1 mclaughlin multistate bar preparation course class 5 property i questions 1-51 copyright 2003...

235

Click here to load reader

Upload: mollie-twiggs

Post on 30-Mar-2015

366 views

Category:

Documents


21 download

TRANSCRIPT

Page 1: 1 McLAUGHLIN MULTISTATE BAR PREPARATION COURSE Class 5 Property I Questions 1-51 Copyright 2003 Walter H. McLaughlin

1

McLAUGHLIN MULTISTATE BAR PREPARATION COURSE

Class 5

Property I

Questions 1-51

• Copyright 2003

• Walter H. McLaughlin

Page 2: 1 McLAUGHLIN MULTISTATE BAR PREPARATION COURSE Class 5 Property I Questions 1-51 Copyright 2003 Walter H. McLaughlin

2

Classification by Difficulty Property Questions 1-51

• Easy 12

1, 7, 8, 20, 25, 27, 30, 31, 32, 36, 44, and 48

• Medium 16

• 3, 9, 13, 17, 19, 21, 23, 26, 28, 34, 35, 41, 42, 43, 47, and 49

• Hard 23

• 2, 4, 5, 6, 10, 11, 12, 14, 15, 16, 18, 22, 24, 29, 33, 37. 38. 39, 40, 45, 46, 50 and 51

Page 3: 1 McLAUGHLIN MULTISTATE BAR PREPARATION COURSE Class 5 Property I Questions 1-51 Copyright 2003 Walter H. McLaughlin

3

Governing Law• A conveyance from owner to “A” or to “A and his

heirs” creates a fee simple interest in A and no interest in the heirs.

Page 4: 1 McLAUGHLIN MULTISTATE BAR PREPARATION COURSE Class 5 Property I Questions 1-51 Copyright 2003 Walter H. McLaughlin

4

Question 1 Degree of Difficulty Easy• July 1998 Q 187• Six years ago, Oscar, owner of Blackacre in fee

simple, executed an instrument in the proper form of a warranty deed, purporting to convey Blackacre to "Albert and his heirs." At that time, Albert was a widower who had one child, Donna.

• Three years ago, Albert executed and delivered to Bea an instrument in the proper form of a warranty deed, purporting to convey Blackacre to “Bea”. Donna did not join in the deed. Bea was and still is unmarried and childless.

• The only possibly applicable statute in the jurisdiction states that any deed will be construed to convey the grantor's entire estate unless expressly limited.

Page 5: 1 McLAUGHLIN MULTISTATE BAR PREPARATION COURSE Class 5 Property I Questions 1-51 Copyright 2003 Walter H. McLaughlin

5

• Last month, Albert died, never having remarried. Donna is his only heir.

• Blackacre is now owned by• (A) Donna, because Albert's death ended

Bea's life estate per autre vie.• (B) Bea in fee simple pursuant to Albert's

deed. • (C) Donna and Bea as tenants-in-common of

equal shares. • (D) Donna and Bea as joint tenants, because

both survived Albert.

Page 6: 1 McLAUGHLIN MULTISTATE BAR PREPARATION COURSE Class 5 Property I Questions 1-51 Copyright 2003 Walter H. McLaughlin

6

C, D and A Are Wrong; B Is Right• C D The words “and his heirs” are words of limitation

meaning to convey a fee simple. They are not words of purchase and create neither a tenancy in common nor a joint tenancy in Donna.

• A Albert had a fee simple, not a life estate and the conveyance to Bea was a conveyance in fee simple

• B Although an old-fashioned form of conveyance, the deed from Oscar to Albert, where the grantee was “Albert and his heirs” conveyed a fee simple absolute to Albert. Albert’s subsequent deed to “Bea” conveyed the fee simple which Albert received to Bea, even though the grantee was not to “Bea and her heirs”.

Page 7: 1 McLAUGHLIN MULTISTATE BAR PREPARATION COURSE Class 5 Property I Questions 1-51 Copyright 2003 Walter H. McLaughlin

7

Governing Law Fee Simple Determinable-Possibility of Reverter

• A fee simple determinable is a created by a conveyance which limits the fee in the terms of the conveyance with words such as “so long as”. The interest in the grantor following a fee simple determinable is a possibility of reverter, a reversionary interest not subject to the rule against perpetuities. However, a right over in a third person after the termination of a fee simple determinable creates an executory interest which is subject to the rule against perpetuities.

Page 8: 1 McLAUGHLIN MULTISTATE BAR PREPARATION COURSE Class 5 Property I Questions 1-51 Copyright 2003 Walter H. McLaughlin

8

Governing Law Fee Simple Subject to a Condition Subsequent-Right of Entry for

Condition Broken• A fee simple subject of a right of entry for condition

broken is created by a conveyance which grants a fee simple interest and then terminates it if a contingency occurs usually with the words “but if”. The reversionary type interest in the grantor is called a “right of entry for condition broken”, and is not subject to the rule against perpetuities. Again if the right of entry is given to a person other than the grantor, that right is an executory interest subject to the rule against perpetuities

Page 9: 1 McLAUGHLIN MULTISTATE BAR PREPARATION COURSE Class 5 Property I Questions 1-51 Copyright 2003 Walter H. McLaughlin

9

Question 2 Degree of Difficulty Hard • Alice conveyed Twinoaks Farm “to Barbara, her

heirs and assigns, so long as the premises are used for residential and farm purposes, then to Charles and his heirs and assigns.” The jurisdiction in which Twinoaks Farm is located has adopted the common-law Rule Against Perpetuities unmodified by statute. As a consequence of the conveyance, Alice's interest in Twinoaks Farm is

• (A) nothing. • (B) a possibility of reverter. • (C) a right of entry for condition broken. • (D) a reversion in fee simple absolute.

Page 10: 1 McLAUGHLIN MULTISTATE BAR PREPARATION COURSE Class 5 Property I Questions 1-51 Copyright 2003 Walter H. McLaughlin

10

A, C and D Are Wrong• A Alice has more than nothing. A possibility of

reverter follows a fee simple determinable when the executory interest to a third party, Charles, is void.

• C A right of entry is the interest retained by the grantor after a fee simple subject to a condition subsequent, not a fee simple determinable.

• D The correct name of Alice’s right is a possibility of reverter. This choice incorrectly states the name of the interest in the grantor after a fee simple determinable.

Page 11: 1 McLAUGHLIN MULTISTATE BAR PREPARATION COURSE Class 5 Property I Questions 1-51 Copyright 2003 Walter H. McLaughlin

11

B Is Right• The interest in Charles and his heirs is an invalid executory

interest, since there is no assurance that it must vest within lives in being plus 21 years. Vesting in this case means that the time at which Charles would be entitled to his interest because the property was no longer used for farming. It therefore violates the Rule Against Perpetuities and the instrument is construed with the language which created that interest deleted. Therefore, the conveyance should read "to Barbara and her heirs and assigns so long as the premises are used for residential and farm purposes." Such a conveyance is a fee simple determinable. Alice, as the grantor, has a possibility of reverter, the interest which follows a fee simple determinable. It is valid; an interest retained in the grantor does not violate the Rule Against Perpetuities.

Page 12: 1 McLAUGHLIN MULTISTATE BAR PREPARATION COURSE Class 5 Property I Questions 1-51 Copyright 2003 Walter H. McLaughlin

12

Governing Law Distinction Between a Conditional Fee and a Covenant

• Reversionary type interests which are not subject to the rule against perpetuities. Holders of these interests after conditional fees are created have the right to regain possession of the land when the condition is violated. Courts will construe language which is ambiguous and does not clearly follow the form of a conditional fee as covenants which the grantor can enforce with equitable remedies such as injunctions rather than the draconian remedy of forfeiture of the grantee’s interest.

Page 13: 1 McLAUGHLIN MULTISTATE BAR PREPARATION COURSE Class 5 Property I Questions 1-51 Copyright 2003 Walter H. McLaughlin

13

Question 3 Degree of Difficulty Medium• July 1991 Q 172• Test owned Blackacre, a vacant one-acre tract of land in State. Five

years ago, he executed a deed conveying Blackacre to "Church for the purpose of erecting a church building thereon." Three years ago, Test died leaving Sonny as his sole heir at law. His duly probated will left "all my Estate, both real and personal, to my friend, Fanny."

• Church never constructed a church building on Blackacre and last month Church, for a valid consideration, conveyed Blackacre to Developer.

• Developer brought an appropriate action to quiet title against Sonny, Fanny, and Church, and joined the appropriate state official. Such official asserted that the charitable trust was created which has not terminated.

• In such action, the court should find that title is now in• (A) Developer. • (B) Sonny. • (C) Fanny. • (D) the state official.

Page 14: 1 McLAUGHLIN MULTISTATE BAR PREPARATION COURSE Class 5 Property I Questions 1-51 Copyright 2003 Walter H. McLaughlin

14

B, C and D Are Wrong; A Is Right• B C The language in the Test-Church deed is that of a

covenant, not a conditional fee and therefore the only property interest created in the grantor is one to enforce an equitable servitude, not a reversionary type interest. If a reversionary right were created, Fanny, Test’s devisee would take in preference to Sonny, Test’s heir because such reversionary interests are devisable.

• D The state official could only prevail if the language of the conveyance set up a charitable trust. Which is not present in the language of the Test-Church deed. At most, it creates a contract, not a charitable trust.

• A While the language of the deed may have created a covenant between Test and the church, the language is not precise enough to create either a fee simple determinable or a fee simple subject to a condition subsequent which would revest title in Test’s devisee if the condition was broken. Title is therefore in developer.

Page 15: 1 McLAUGHLIN MULTISTATE BAR PREPARATION COURSE Class 5 Property I Questions 1-51 Copyright 2003 Walter H. McLaughlin

15

Governing Law Waste

• The holder of an interest after a qualified fee simple determinable or fee simple subject to a condition subsequent cannot enjoin the holder of the fee simple for waste.

Page 16: 1 McLAUGHLIN MULTISTATE BAR PREPARATION COURSE Class 5 Property I Questions 1-51 Copyright 2003 Walter H. McLaughlin

16

Question 4 Degree of Difficulty Hard• Otto conveyed Goldacre to "Andy, his heirs and assigns, but if Andy

dies and is not survived by children by his present wife, Jane, then to Bob and his heirs and assigns. Shortly after taking possession, Andy discovered rich metal deposits on the land, opened a mining operation, and removed and sold a considerable quantity of valuable ore without giving Bob any notice of his action. Andy has no children. Andy, Jane, and Bob are all still living. Bob brought an action in equity for an accounting of the value of the ore removed and for an injunction against further removal.

• If the decision is for Andy, it will be because• (A) Bob has no interest in Goldacre. • (B) the right to take minerals is an incident of a defeasible fee

simple. • (C) the right to take minerals is an incident of the right to

possession. • (D) there was no showing that Andy acted in bad faith.

Page 17: 1 McLAUGHLIN MULTISTATE BAR PREPARATION COURSE Class 5 Property I Questions 1-51 Copyright 2003 Walter H. McLaughlin

17

A, C and D Are Wrong• A Until Andy dies survived by children of himself and Jane,

Bob has a contingent executory interest in the property known as a shifting use. That use will become possessory in Bob at Andy’s death if Andy dies without such children surviving Andy. Unlike the earlier question this executory interest after a fee simple does not violate the rule against perpetuities because it will vest in possession if it is ever to vest at the time of the death of a life in being, namely. Andy.

• C This statement is too broad. The right to take minerals is not an incident to a right to possession when there is a holder of a subsequent interest such as a remainderman after a life estate. In that case the mining operation would constitute waste which could be enjoined by the remainderman.

• D This issue is whether Bob has a legal right to enjoin waste. Andy’s good faith is irrelevant.

Page 18: 1 McLAUGHLIN MULTISTATE BAR PREPARATION COURSE Class 5 Property I Questions 1-51 Copyright 2003 Walter H. McLaughlin

18

B Is Right• The holder of a remainder interest in property after

interests such aS life estates, has a right to prevent the holder of the present possessory interest from committing waste by such actions as depleting the mineral resources on the property. However, in this case Andy was granted a fee simple interest subject to defeasance if he dies and is not survived by children of his present wife, Jane. If he dies survived by children of himself and Jane, the interest in his estate is a fee simple absolute which he can devise to anyone. Therefore, there is a high likelihood that courts would not apply the doctrine of waste in this case.

Page 19: 1 McLAUGHLIN MULTISTATE BAR PREPARATION COURSE Class 5 Property I Questions 1-51 Copyright 2003 Walter H. McLaughlin

19

Governing Law Remainders• A remainder is a future interest which is created in a

party other than the grantor in the same instrument as the prior possessory interest and which is capable of taking effect at the termination of the prior interest.

• A remainder is contingent if there is a condition precedent which must be satisfied before the interest becomes possessory or if the holders of the interest such as heirs of a living person or unborn children, are unascertained.

• A remainder is vested if it is ready to take in possession whenever and however the previous possessory interest terminates.

Page 20: 1 McLAUGHLIN MULTISTATE BAR PREPARATION COURSE Class 5 Property I Questions 1-51 Copyright 2003 Walter H. McLaughlin

20

Question 5 Degree of Difficulty Hard• July 1998 Q 181• Theresa owned Blueacre, a tract of land, in fee simple.

Theresa wrote and executed, with the required formalities, a will that devised Blueacre to "my daughter, Della, for life with remainder to my descendants per stirpes." At the time of writing the will, Theresa had a husband and no descendants living other than her two children, Della and Seth.

• Theresa died and the will was duly admitted to probate. Theresa's husband predeceased her.

• Theresa was survived by Della, Seth, four grandchildren, and one great-grandchild.

• Della and Seth were Theresa's sole heirs at law.

Page 21: 1 McLAUGHLIN MULTISTATE BAR PREPARATION COURSE Class 5 Property I Questions 1-51 Copyright 2003 Walter H. McLaughlin

21

• Della and Seth brought an appropriate action for declaratory judgment as to title of Blueacre. Guardians ad litem were appointed and all other steps were taken so that the judgment would bind all persons interested, whether born or unborn.

• In that action, if the court rules that Della has a life estate in the whole of Blueacre and that the remainder is contingent, it will be because the court chose one of several possible constructions and that the chosen construction

• (A) related all vesting to the time of writing of the will. • (B) related all vesting to the death of Theresa. • (C) implied a condition that remaindermen survive Della. • (D) implied a gift of a life estate to Seth.

Page 22: 1 McLAUGHLIN MULTISTATE BAR PREPARATION COURSE Class 5 Property I Questions 1-51 Copyright 2003 Walter H. McLaughlin

22

A, B and D Are Wrong• A B If all vesting related to the time of the writing of the

will or at the time of Theresa’s death, then there would be no contingent remainders because the persons who would take per stirpes would be determined at the time specified, and there would be no condition precedent to anyone taking. For example, if vesting occurred at the death of Theresa, Seth would have a vested remainder in one-half of Blackacre and the children of Della then living would have a remainder in the other half as tenants-in-common.

• D Even if Seth had a life estate after Della’s life estate, the interest of the remaindermen could still be vested at the time of Teresa’s death, and there would be no contingent remainders.

Page 23: 1 McLAUGHLIN MULTISTATE BAR PREPARATION COURSE Class 5 Property I Questions 1-51 Copyright 2003 Walter H. McLaughlin

23

C Is Right• If the court rules that Della has a life estate in the whole of

Blueacre and that the remainder is contingent, it is because there is a condition precedent which must be satisfied before the remaindermen can take. That condition precedent would have to be that the persons who are to take per stirpes must survive Della. For example, at Theresa’s death, Della’s children would be the persons who would take one- half of the remainder, and Seth would take the other half. If this construction is followed, the persons who take per stirpes would be required to survive Della in order to take. If Seth predeceased Della, the one-half interest belonging to Seth’s side of the family would go to his issue per stirpes rather than be disposed of in accordance with Seth’s will.

Page 24: 1 McLAUGHLIN MULTISTATE BAR PREPARATION COURSE Class 5 Property I Questions 1-51 Copyright 2003 Walter H. McLaughlin

24

Question 6 Degree of Difficulty Hard• Martinez, a widower, owns in fee simple a ranch,

Ranchacre. Martinez has one child, Enrique, who is married. Enrique has one child, Ana Maria, who is also married but has no children. In an effort to dispose of Ranchacre to his descendants and to honor a request by Ana Maria that she be skipped in any disposition, Martinez conveys Ranchacre to his son, Enrique, for life with the remainder to Ana Maria's children in fee simple.

• What interest, if any, is created in favor of Ana Maria's unborn children at the time of the conveyance?

• (A) A contingent remainder• (B) A vested remainder subject to divestment• (C) A springing use• (D) None

Page 25: 1 McLAUGHLIN MULTISTATE BAR PREPARATION COURSE Class 5 Property I Questions 1-51 Copyright 2003 Walter H. McLaughlin

25

C and D Are Wrong• C If Enrique dies before Ana Maria has children,

then the property will revert to the grantor, Martinez, at Enrique's death, until children are born to Ana Maria. In the meantime, the unborn children will no longer hold a contingent remainder, but will hold a springing use. However, until Enrique dies, the unborn children of Ana Maria hold a contingent remainder.

• D The interest in the children of Ana Maria does not violate the Rule Against Perpetuities because their interest will vest at the latest at death of a life in being, Ana Maria. All of Ana Maria’s children must be born during her lifetime.

Page 26: 1 McLAUGHLIN MULTISTATE BAR PREPARATION COURSE Class 5 Property I Questions 1-51 Copyright 2003 Walter H. McLaughlin

26

B Is Wrong and A Is Right• The interest in the unborn children is a remainder interest

since the interest was created in the same conveyance as a present possessory interest and is intended to become possessory at the expiration of the present possessory interest. The remainder is contingent because there are no ascertained individuals capable of taking at the time of the conveyance, since Ana Maria has not yet had a child. As soon as a child of Ana Maria is born, the interest in that child becomes a vested remainder which is subject to open in the event that other children of Ana Maria are born. It is never subject to divestment since there are no conditions which will terminate the vested remainder.

Page 27: 1 McLAUGHLIN MULTISTATE BAR PREPARATION COURSE Class 5 Property I Questions 1-51 Copyright 2003 Walter H. McLaughlin

27

Governing Law Life Estate

• At the termination of the measuring life of a life estate, the property reverts to the grantor unless the grantor has also deeded a remainder interest after the life estate, in which case title then vests in the remainderman

• If a person holds a life estate per autre vie (that is the measuring life is another person), the estate terminates at the death of the person who is the measuring life, and not upon the death of the person in possession.

Page 28: 1 McLAUGHLIN MULTISTATE BAR PREPARATION COURSE Class 5 Property I Questions 1-51 Copyright 2003 Walter H. McLaughlin

28

Question 7 Degree of Difficulty Easy• July 1991 Q 92• Ogle owned Greenacre, a tract of land, in fee simple. Five

years ago, he executed and delivered to Lilly an instrument in the proper form of a warranty deed that conveyed Greenacre to Lilly "for and during the term of her natural life." No other estate or interest or person taking an interest was mentioned. Lilly took possession of Greenacre and has remained in possession.

• Fifteen months ago, Ogle died, leaving a will that has been duly admitted to probate. The will, inter alia, had the following provision:– “I devise Greenacre to Mina for her natural life and from

and after Mina's death to Rex, his heirs and assigns, forever.”

Page 29: 1 McLAUGHLIN MULTISTATE BAR PREPARATION COURSE Class 5 Property I Questions 1-51 Copyright 2003 Walter H. McLaughlin

29

• Administration of Ogle's estate has been completed. Mina claims the immediate right to possession of Greenacre. Rex also asserts a right to immediate possession.

• In an appropriate lawsuit to which Lilly, Mina, and Rex are parties, who should be adjudged to have the right to immediate possession?

• (A) Lilly, because no subsequent act of Ogle would affect her life estate.

• (B) Mina, because Ogle's will was the final and definitive expression of his intent.

• (C) Mina, because Lilly's estate terminated with the death of Ogle.

• (D) Rex, because Lilly's estate terminated with Ogle's death and all that Ogle had was the right to transfer his reversion in fee simple.

Page 30: 1 McLAUGHLIN MULTISTATE BAR PREPARATION COURSE Class 5 Property I Questions 1-51 Copyright 2003 Walter H. McLaughlin

30

B, C and D Are Wrong; A Is Right• B While Ogle’s will was the final expression of his intent, it can only

affect what he owns. He owns a reversion which becomes possessory after Mina dies. Mina’s life estate which is measured by Mina’s life will become possessory at Lilly’s death if Mina survives Lilly.

• C Lilly’s life not Ogle’s life is the measuring life for Lilly’s life estate.

• D Lilly’s life not Ogle’s life is the measuring life for Lilly’s life estate. Therefore, the reversion to Rex only becomes possessory after the termination of Lilly’s life estate and then Mina’s life estate which Ogle carved out of his reversion.

• A After the conveyance of the life estate to Lilly, Ogle retained a reversion which would take effect in possession at the termination of Lilly’s life estate at Lilly’s death. Since Lilly is still alive, her life estate is still valid and she is entitled to possession until her death.

Page 31: 1 McLAUGHLIN MULTISTATE BAR PREPARATION COURSE Class 5 Property I Questions 1-51 Copyright 2003 Walter H. McLaughlin

31

Question 8 Degree of Difficulty Easy• July 1991 Q 138 • Ody, owner of Profitacre executed an instrument in the

proper form of a deed, purporting to convey Profitacre "to Leon for life, then to Ralph in fee simple." Leon, who is Ody's brother and Ralph's father, promptly began to manage Profitacre, which is valuable income-producing real estate. Leon collected all rents and paid all expenses, including real estate taxes. Ralph did not object, and this state of affairs continued for five years until 1987. In that year, Leon executed an instrument in the proper form of a deed, purporting to convey Profitacre to Mona. Ralph, no admirer of Mona, asserted his right to ownership of Profitacre. Mona asserted her ownership and said that if Ralph had any rights, he was obligated to pay real estate taxes, even though Leon had been kind enough to pay them in the past.

Page 32: 1 McLAUGHLIN MULTISTATE BAR PREPARATION COURSE Class 5 Property I Questions 1-51 Copyright 2003 Walter H. McLaughlin

32

• Income from Profitacre is ample to cover expenses, including real estate taxes.

• In an appropriate action to determine the rights of the parties, the court should decide

• (A) Leon's purported deed forfeited his life estate, so Ralph owns Profitacre in fee simple.

• (B) Mona owns an estate for her life, is entitled to all income, and must pay real estate taxes; Ralph owns the remainder interest.

• (C) Mona owns an estate for the life of Leon, is entitled to all income, and must pay real estate taxes; Ralph owns the remainder interest.

• (D) Mona owns an estate for the life of Leon and is entitled to all income; Ralph owns the remainder interest, and must pay real estate taxes.

Page 33: 1 McLAUGHLIN MULTISTATE BAR PREPARATION COURSE Class 5 Property I Questions 1-51 Copyright 2003 Walter H. McLaughlin

33

A, B and D Are Wrong• A Leone’s purported deed does not forfeit his life

estate. It merely conveys what he has, the balance of the life estate, to Mona. Mona now has a life estate per autre vie.

• B The measuring life state is wrong. The correct measure for the life estate owned by Mona is Leon’s life, not Mona’s life.

• D While Mona may collect the rent as the life tenant per autre vie, as the owner of the present possessory interest, she also has the obligation to pay the current real estate taxes during the term of her life estate.

Page 34: 1 McLAUGHLIN MULTISTATE BAR PREPARATION COURSE Class 5 Property I Questions 1-51 Copyright 2003 Walter H. McLaughlin

34

C Is Right• When Leon received a life estate from Ody, he had

the right to possession of the property, the right to collect the rents and the obligation to pay the current expenses associated with those rents, namely, the property taxes. The conveyance of Profitacre from Leon to Mona was only a conveyance of what Leon had, the balance of his life estate. Mona, for the remaining life of Leon, has the right to collect rents and the obligation to pay taxes. Ralph has a vested remainder interest which becomes possessory at the death of Leon.

Page 35: 1 McLAUGHLIN MULTISTATE BAR PREPARATION COURSE Class 5 Property I Questions 1-51 Copyright 2003 Walter H. McLaughlin

35

Governing Law Waste

• Waste is an action of the life tenant which reduces the value of the future interest in the property. However if the life tenant takes an action which increase the value of the remainder, that action is ameliorating waste and the life tenant is not liable.

• Any remainderman (including a contingent remainderman) can enjoin a life tenant from committing waste.

Page 36: 1 McLAUGHLIN MULTISTATE BAR PREPARATION COURSE Class 5 Property I Questions 1-51 Copyright 2003 Walter H. McLaughlin

36

Question 9 Degree of Difficulty Medium• July 1991 Q 179• Alice owned a commercial property, Eastgate, consisting of a one-

story building rented to various retail stores and a very large parking lot. Two years ago, Alice died and left Eastgate to her nephew, Paul, for life, with remainder to her godson, Richard, his heirs and assigns. Paul was 30 years old and Richard was 20 years old when Alice died. The devise of Eastgate was made subject to any mortgage on Eastgate in effect at the time of Alice's death.

• When Alice executed her will, the balance of the mortgage debt on Eastgate was less than $5,000. A year before her death, Alice suffered financial reverses, and in order to meet her debts, she had mortgaged Eastgate to secure a loan of $150,000. The entire principal of the mortgage remained outstanding when she died. As a result, the net annual income from Eastgate was reduced not only by real estate taxes and regular maintenance costs, but also by the substantial mortgage interest payments that were due each month. Paul was very dissatisfied with the limited benefit that he was receiving from the life estate.

Page 37: 1 McLAUGHLIN MULTISTATE BAR PREPARATION COURSE Class 5 Property I Questions 1-51 Copyright 2003 Walter H. McLaughlin

37

• When, earlier this year, Acme, Inc. proposed to purchase Eastgate, demolish the building, pay off the mortgage, and construct a 30-story office building, Paul was willing to accept Acme's offer.

• However, Richard adamantly refused the offer, even though Richard, as the remainderman, paid the principal portion of each monthly mortgage amortization payment. Richard was independently wealthy and wanted to convert Eastgate into a public park when he became entitled to possession.

• When Acme realized that Richard would not change his mind, Acme modified its proposal to a purchase of the life estate of Paul. Acme was ready to go ahead with its building plans, relying upon a large life insurance policy on Paul's life to protect it against the economic risk of Paul's death. Paul's life expectancy was 45 years.

Page 38: 1 McLAUGHLIN MULTISTATE BAR PREPARATION COURSE Class 5 Property I Questions 1-51 Copyright 2003 Walter H. McLaughlin

38

• When Richard learned that Paul had agreed to Acme's modified proposal, Richard brought an appropriate action against them to enjoin their carrying it out.

• There is no applicable statute. • The best argument for Richard is that• (A) Acme cannot purchase Paul's life estate, because life

estates are not assignable• (B) the proposed demolition of the building constitutes

waste. • (C) Richard's payment of the mortgage principal has

subrogated him to Paul's rights as a life tenant and bars Paul's assignment of the life estate without Richard's consent.

• (D) continued existence of the one-story building is more in harmony with the ultimate use as a park than the proposed change in use.

Page 39: 1 McLAUGHLIN MULTISTATE BAR PREPARATION COURSE Class 5 Property I Questions 1-51 Copyright 2003 Walter H. McLaughlin

39

A, C and D Are Wrong; B Is Right• A A life estate is an alienable interest in property.• C Richard’s payment of the mortgage principal is an independent

obligation of Richard as remainderman and subrogates him to nothing. When he obtains possession of the property after the termination of the life estate, the mortgage principal will have been reduced by his payments.

• D The remainderman’s ultimate uneconomic use of the property is irrelevant to the determination that destruction of the building is waste. Richard can tear down the office building once he is entitled to possession.

• B A remainderman ordinarily has the right to prevent the life tenant from destroying an income producing building on the property because it would constitute waste. While it is a close question of whether the action improves the property by building a large office building on the site is ameliorating waste, which he could not prevent, the doctrine of waste is Richard’s best argument for an injunction.

Page 40: 1 McLAUGHLIN MULTISTATE BAR PREPARATION COURSE Class 5 Property I Questions 1-51 Copyright 2003 Walter H. McLaughlin

40

Governing Law Life Estates• If there is a mortgage on property at the time it is

conveyed to a life tenant and a remainderman and there is income generated by the property, the life tenant is responsible for interest and current real estate taxes, and the remainderman is responsible for paying the principal. Neither life tenant nor remainderman is personally liable on the mortgage note. If there is no income generated by the property, the life tenant in not personally liable to the remainderman for interest and taxes paid by him.

Page 41: 1 McLAUGHLIN MULTISTATE BAR PREPARATION COURSE Class 5 Property I Questions 1-51 Copyright 2003 Walter H. McLaughlin

41

Question 10 Degree of Difficulty Hard• July 1998 Q 75• Otis owned in fee simple Lots 1 and 2 in an urban

subdivision. The lots were vacant and unproductive. They were held as a speculation that their value would increase. Otis died and, by his duly probated will, devised the residue of his estate (of which Lots 1 and 2 were part) to Lena for life with remainder in fee simple to Rose. Otis's executor distributed the estate under appropriate court order, and notified Lena that future real estate taxes on Lots 1 and 2 were Lena's responsibility to pay.

• Except for the statutes relating to probate and those relating to real estate taxes, there is no applicable statute.

Page 42: 1 McLAUGHLIN MULTISTATE BAR PREPARATION COURSE Class 5 Property I Questions 1-51 Copyright 2003 Walter H. McLaughlin

42

• Lena failed to pay the real estate taxes due for Lots 1 and 2. To prevent a tax sale of the fee simple, Rose paid the taxes and demanded that Lena reimburse her for same. When Lena refused, Rose brought an appropriate action against Lena to recover the amount paid.

• In such action, Rose should recover• (A) the amount paid, because a life tenant has the duty to

pay current charges. • (B) the present value of the interest that the amount paid

would earn during Lena's lifetime. • (C) nothing, because Lena's sole possession gave the

right to decide whether or not taxes should be paid. • (D) nothing, because Lena never received any income

from the lots.

Page 43: 1 McLAUGHLIN MULTISTATE BAR PREPARATION COURSE Class 5 Property I Questions 1-51 Copyright 2003 Walter H. McLaughlin

43

A, B and C Are Wrong• A B A states the usual rule that the life tenant is responsible

for the real estate taxes on the property because they are ordinarily an annual recurring expense which reduces the income the property earns. B does not state a principal which is ever recognized. Both are wrong in this case, however, because the property is unproductive and produces no income. If the property were producing income and the remainderperson paid the taxes, she could recover them from the life tenant to the extent that life tenant received income. However, in this case there is no income and she can recover nothing.

• C Lena does not have the sole right to decide if the taxes should be paid. If the property is lost to foreclosure, Rose will never come into possession of her remainder interest. Therefore, Rose has a right to pay the taxes, but that does not give her the right in this case to recover reimbursement from Lena.

Page 44: 1 McLAUGHLIN MULTISTATE BAR PREPARATION COURSE Class 5 Property I Questions 1-51 Copyright 2003 Walter H. McLaughlin

44

D Is Right• Even though Lena has a present possessory interest the

property, she gets no benefit out of it and may never benefit from her life interest. Therefore, Lena has chosen not to pay the taxes and is willing to lose her interest in the property to tax foreclosure. Rose, on the other hand, will eventually be able to sell the property and realize its long-term value. If she sells it when her interest becomes possessory, then she will owe nothing to the life tenant. Therefore, Rose is a volunteer with respect to the payment of the taxes at a time when there is no income from the property and will not recover either the full amount of the taxes paid, nor the present value of the interest that the amount paid would earn during Lena’s lifetime.

Page 45: 1 McLAUGHLIN MULTISTATE BAR PREPARATION COURSE Class 5 Property I Questions 1-51 Copyright 2003 Walter H. McLaughlin

45

Questions 11 and 12 are based on the following fact situation.

• Trease owned Hilltop in fee simple. By his will, he devised as follows: “Hilltop to such of my grandchildren who shall reach the age of 21; and by this provision I intend to include all grandchildren whenever born.” At the time of his death, Trease had three children and two grandchildren.

Page 46: 1 McLAUGHLIN MULTISTATE BAR PREPARATION COURSE Class 5 Property I Questions 1-51 Copyright 2003 Walter H. McLaughlin

46

Question 11 Degree of Difficulty Hard• Courts hold such a devise valid under the Common-law

Rule Against Perpetuities. What is the best explanation of that determination?

• (A) All of Trease's children would be measuring lives. • (B) The rule of convenience closes the class of

beneficiaries when any grandchild reaches the age of 21.

• (C) There is a presumption that Trease intended to include only those grandchildren born prior to his death.

• (D) There is a subsidiary rule of construction that dispositive instruments are to be interpreted so as to uphold interests rather than to invalidate them under the Rule Against Perpetuities.

Page 47: 1 McLAUGHLIN MULTISTATE BAR PREPARATION COURSE Class 5 Property I Questions 1-51 Copyright 2003 Walter H. McLaughlin

47

Beginning of Time Line• Persons who are measuring lives must be alive when

rule begins to run

• In case of a devise by will, rule begins to run at death of testator.

• All of Trease’s children are lives in being or measuring lives except for deceased children whose children, if any, have to be alive and therefore are measuring lives

Page 48: 1 McLAUGHLIN MULTISTATE BAR PREPARATION COURSE Class 5 Property I Questions 1-51 Copyright 2003 Walter H. McLaughlin

48

End of Time Line• When all interests must vest or interest is invalid.

• In this case it is 21 years after the birth of the youngest grandchild because this is the time when all grandchildren who must be born during lives of Trease’s children have either reached age 21 or are dead.

Page 49: 1 McLAUGHLIN MULTISTATE BAR PREPARATION COURSE Class 5 Property I Questions 1-51 Copyright 2003 Walter H. McLaughlin

49

B and C are Wrong• B The rule of convenience is a rule of construction which,

based on the grantor's presumed intent, closes the class when the first member of the class reaches age 21 and is capable of taking a possessory interest. In this case, though, the facts indicate that Trease intended to include all grandchildren whenever born in the gift. Therefore, the rule of convenience does not apply.

• C There is no presumption that a testator who uses the word "grandchildren" only intends to mean those grandchildren born prior to his death. Even if there were such a presumption, it would be negated by the testator's language which clearly indicates an intent to include all grandchildren whenever born.

Page 50: 1 McLAUGHLIN MULTISTATE BAR PREPARATION COURSE Class 5 Property I Questions 1-51 Copyright 2003 Walter H. McLaughlin

50

D Is Wrong and A Is Right• D At common law, the Rule Against Perpetuities was applied with

rigor, and there was no rule of construction which was used to save gifts which violated the Rule. Moreover, the interests created in this case do not violate the Rule Against Perpetuities and need no rule of construction to save them. Also, as a general principle, this type of vague, policy-based answer should be avoided, especially in an area like the Rule Against Perpetuities, where the Examiners expect a detailed knowledge of the law and a detailed analysis.

• A The will does not become effective until Trease's death and so it does not create any interests until that time. Therefore, the Rule period is measured from Trease's death. At that time, all of Trease's children must be alive. The children thus may act as the measuring lives. The interests in the grandchildren, whenever born, must vest, if they are ever to vest, within 21 years of the death of the last measuring life. Again, by definition, the last grandchild must be born by the end of the last child's life. Therefore, this interest, which vests when each grandchild reaches the age of 21, must vest within 21 years of the death of the last measuring life, a child of Trease.

Page 51: 1 McLAUGHLIN MULTISTATE BAR PREPARATION COURSE Class 5 Property I Questions 1-51 Copyright 2003 Walter H. McLaughlin

51

Question 12 Degree of Difficulty Hard• Which of the following additions to or changes in

the facts of the preceding question would produce a violation of the common-law Rule Against Perpetuities?

• (A) A posthumous child was born to Trease.

• (B) Trease's will expressed the intention to include all afterborn grandchildren in the gift.

• (C) The instrument was an inter vivos conveyance rather than a will.

• (D) Trease had no grandchildren living at the time of his death.

Page 52: 1 McLAUGHLIN MULTISTATE BAR PREPARATION COURSE Class 5 Property I Questions 1-51 Copyright 2003 Walter H. McLaughlin

52

A and B Are Wrong• A The fact that a child was born to Trease after his death would not

invalidate the disposition under the Rule Against Perpetuities because all children conceived at the time of a person's death are eligible to be treated as lives in being at the time of the testator's death for the purpose of computing the time limits under the Rule. Therefore, an afterborn child is a life in being to the same extent as a child born before the death of Trease.

• B This is a choice which can be eliminated even if your knowledge of perpetuities is limited. The focus of the question asks what additions or changes in the preceding paragraph will cause a violation of the Rule. This choice does not represent an addition or change. Trease's will already expresses the intent to include afterborn grandchildren. That fact did not cause a violation of the Rule. It is important in answering these questions to read them carefully and always compare the choices to the facts of the question.

Page 53: 1 McLAUGHLIN MULTISTATE BAR PREPARATION COURSE Class 5 Property I Questions 1-51 Copyright 2003 Walter H. McLaughlin

53

D Is Wrong • The fact that Trease had no grandchildren alive at his death

is not relevant, because the grandchildren are not the measuring lives, Trease's children are. Since the will does not become effective until Trease's death, the period of the rule is measured from Trease's death. At that time, all of Trease's children must either be alive or have predeceased Trease leaving children who are alive. These children, who must be born before Trease’s death, may thus act as measuring lives. The interests of the grandchildren, which vest when each grandchild reaches the age of 21, must vest within 21 years of the death of the last measuring life, a child of Trease or a grandchild of a deceased child of Trease who was born before Trease’s death.

Page 54: 1 McLAUGHLIN MULTISTATE BAR PREPARATION COURSE Class 5 Property I Questions 1-51 Copyright 2003 Walter H. McLaughlin

54

C Is Right• If this grant were made inter vivos, the time for the running

of the Rule would commence at the date of the gift, rather than at the death of Trease. It is possible that another child could be born to Trease after the date of the gift who would then not be a life in being for purposes of computing the time limits under the Rule Against Perpetuities. It is possible that the child born after the date of the deed would have a child whose interest would still be contingent because the child had not yet reached age 21 at the death of the last of Trease's children alive at the date of the deed. Because that child would be a member of the class of grandchildren, the gift would be invalid at common law as to all members of the class.

Page 55: 1 McLAUGHLIN MULTISTATE BAR PREPARATION COURSE Class 5 Property I Questions 1-51 Copyright 2003 Walter H. McLaughlin

55

Governing Law Contingent Remainders• A remainder is a future interest which is created in a

party other than the grantor in the same instrument as the prior possessory interest and which is capable of taking effect at the termination of the prior interest.

• A remainder is contingent if there is a condition precedent which must be satisfied before the interest becomes possessory or if the holders of the interest such as heirs of a living person or unborn children, are unascertained.

Page 56: 1 McLAUGHLIN MULTISTATE BAR PREPARATION COURSE Class 5 Property I Questions 1-51 Copyright 2003 Walter H. McLaughlin

56

Question 13 Degree of Difficulty Medium• July 1991 Q 187• Oren owned Purpleacre, a tract of land, in fee simple. By

will duly admitted to probate after his death, Oren devised Purpleacre to “any wife who survives me with remainder to such of my children as are living at her death.”

• Oren was survived by Wen, his wife, and by three children, Cynthia, Cam, and Camelia. Thereafter, Cam died and by will duly admitted to probate devised his entire estate to David. Cynthia and Camelia were Cam's heirs at law.

• Later Wen died. In an appropriate lawsuit to which Cynthia, Camelia, and David are parties, title to Purpleacre is at issue.

Page 57: 1 McLAUGHLIN MULTISTATE BAR PREPARATION COURSE Class 5 Property I Questions 1-51 Copyright 2003 Walter H. McLaughlin

57

• In such lawsuit, judgment should be that title to Purpleacre is in

• (A) Cynthia, Camelia, and David, because the earliest vesting of remainders is favored and reference to Wen's death should be construed as relating to time of taking possession.

• (B) Cynthia, Camelia, and David, because the provision requiring survival of children violates the Rule

Against Perpetuities since the surviving wife might have been a person unborn at the time of writing of the will.

• (C) Cynthia and Camelia, because Cam's remainder must descend by intestacy and is not devisable.

• (D) Cynthia and Camelia, because the remainders were contingent upon surviving the life tenant.

Page 58: 1 McLAUGHLIN MULTISTATE BAR PREPARATION COURSE Class 5 Property I Questions 1-51 Copyright 2003 Walter H. McLaughlin

58

A, B and C Are Wrong; D Is Right• A The language of the grant of the remainder interest is to “such of

my children who are living at her death”. This language clearly creates a condition precedent of survivorship. Therefore, Cam’s devisee, David, does not take.

• B Even if the surviving wife could be a person unborn at the time of the writing of the will, the critical time to determine lives in being for purposes of the rule against perpetuities is at Oren’s death. Wen and all three children were lives in being at that time. There is no violation of the rule against perpetuities.

• C The remainder interest was validly created by devise in Wen’s will. Cynthia and Camilla take by grant under the will, not by intestacy from Cam.

• D Cynthia, Cam, and Camilla, held remainder interests in Purpleacre which were contingent upon each of them surviving the life tenant Wen. Since Cam did not survive, his contingent remainder is invalid because he did not satisfy the condition precedent by surviving Wen. Cynthia and Camilla are the co-owners of the property because they satisfied the condition precedent by surviving the life tenant.

Page 59: 1 McLAUGHLIN MULTISTATE BAR PREPARATION COURSE Class 5 Property I Questions 1-51 Copyright 2003 Walter H. McLaughlin

59

Question 14 Degree of Difficulty Hard• July 1991 Q 64.• Sixty years ago by a properly executed and recorded deed, Albert

conveyed Greenacre, a tract of land: "To Louis for life, then to Louis' widow for her life, then to Louis' child or children in equal shares." At that time, Louis, who was Albert's grandson, was six years old.

• Shortly thereafter, Albert died testate. Louis was his only heir at law. Albert's will left his entire estate to First Church.

• Twenty-five years ago, when he was 41, Louis married Maria who was then 20 years old; they had one child, Norman. Maria and Norman were killed in an automobile accident three years ago when Norman was 21. Norman died testate, leaving his entire estate to the American Red Cross. His father, Louis, was Norman's sole heir at law.

• Two year ago, Louis married Zelda. They had no children. This year, Louis died testate, survived by his widow, Zelda, to whom he left his entire estate.

• The common-law Rule Against Perpetuities is unchanged by statute in the jurisdiction.

Page 60: 1 McLAUGHLIN MULTISTATE BAR PREPARATION COURSE Class 5 Property I Questions 1-51 Copyright 2003 Walter H. McLaughlin

60

• In an appropriate action to determine the ownership of Greenacre, the court should find that title is vested in

• (A) First Church, because the widow of Louis was unborn at the time of conveyance and, hence, the remainder violated the Rule Against Perpetuities.

• (B) Zelda, because her life estate and her inheritance from Louis (who was Albert's sole heir at law and who was Norman's sole heir at law) merged the entire title in

her.

• (C) the American Red Cross, because Norman had a vested remainder interest (as the only child of Louis) that it inherited, the life estate to Louis' widow being of no

force and effect.

• (D) Zelda for life under the terms of Albert's deed, with the remainder to the American Red Cross as the successor in interest to Norman, Louis' only child.

Page 61: 1 McLAUGHLIN MULTISTATE BAR PREPARATION COURSE Class 5 Property I Questions 1-51 Copyright 2003 Walter H. McLaughlin

61

Time Line1925 Louis, Albert’s grandson born1931 Albert executes deed to Greenacre1932 Albert dies testate; Louis sole heir at law; First Church sole devisee.1946 Louis’s first wife, Maria, born1966 Louis marries Maria.1967 Louis’s son, Norman, born.1988 Maria and Norman die; Louis is Norman’s sole

heir. American Red Cross Norman’s sole devisee.

1989 Louis marries Zelda.1991 Louis dies. Zelda is the sole beneficiary.

Page 62: 1 McLAUGHLIN MULTISTATE BAR PREPARATION COURSE Class 5 Property I Questions 1-51 Copyright 2003 Walter H. McLaughlin

62

Part I: Since this question was asked on the July 1991 exam all time periods relate to 1991• Facts Sixty years ago by a properly executed and recorded

deed, Albert conveyed Greenacre, a tract of land: "To Louis for life, then to Louis' widow for her life, then to Louis' child or children in equal shares." At that time, Louis, who was Albert's grandson, was six years old.

• Translation • In 1931 Albert deeded a life estate to six year old Louis,

Albert’s grandson, born in 1925, then to Louis’ Widow for life, then to Louis’ child or children in equal shares.Analysis Louis has a life estate, Louis’ widow, who is unascertained, has a contingent remainder in a life estate; Louis child or children, who are unascertained have a contingent remainder in fee. Albert has a reversion if Louis dies without ever having had children.

Page 63: 1 McLAUGHLIN MULTISTATE BAR PREPARATION COURSE Class 5 Property I Questions 1-51 Copyright 2003 Walter H. McLaughlin

63

Part II• Facts Shortly thereafter, Albert died testate. Louis was his

only heir at law. Albert's will left his entire estate to First Church.

• Translation• 1932 Albert dies. Louis is his sole heir. First Church is his

sole devisee.• Analysis• At the time of his death First Church as Albert’s devisee has

a reversion which would become possessory at Louis’ death or in the alternative, at the death of Louis’ widow if Louis dies without ever having had children. Since Louis had a child, Norman, in 1967, Albert’s reversion terminated at that time.

Page 64: 1 McLAUGHLIN MULTISTATE BAR PREPARATION COURSE Class 5 Property I Questions 1-51 Copyright 2003 Walter H. McLaughlin

64

Part III• Facts• Twenty-five years ago, when he was 41, Louis married

Maria, who was then 20 years old; they had one child, Norman.

• Maria and Norman were killed in an automobile accident three years ago when Norman was 21.

• Translation• Maria born in 1946, marries Louis in 1966 and dies in 1988

before Louis dies. • Analysis• Maria has a life estate which is valid under the rule against

perpetuities because it must vest if at all at Louis’ death. It is contingent on Maria surviving Louis and being married to him at his death, Maria’s contingent life estate terminates at her death in 1988.

Page 65: 1 McLAUGHLIN MULTISTATE BAR PREPARATION COURSE Class 5 Property I Questions 1-51 Copyright 2003 Walter H. McLaughlin

65

• Part IV Facts

• [T]hey [Louis and Maria] had one child, Norman. Maria and Norman were killed in an automobile accident three years ago when Norman was 21. Norman died testate, leaving his entire estate to the American Red Cross. His father, Louis, was Norman's sole heir at law.

• Translation 1967 Norman, Louis’ and Maria’s son, is born. He predeceases Louis, dying in 1988.

Page 66: 1 McLAUGHLIN MULTISTATE BAR PREPARATION COURSE Class 5 Property I Questions 1-51 Copyright 2003 Walter H. McLaughlin

66

• Analysis • Norman has a vested remainder, which becomes possessory

at the termination of the life estate to Louis and the possible life estate to Louis’ widow, subject to open if Louis has any more children during his lifetime. The remainder is vested because he is ascertained and there is no condition precedent to his taking. Therefore, Norman’s death before Louis’s death does not defeat his interest. However, any additional children of Louis would share in the gift. There is no violation of the rule against perpetuities. The interest vests at his birth which is within the lifetime of Louis, a life in being at the time of the creation of the transfer in 1931. Norman’s entire estate goes to the American Red Cross. Louis takes nothing from Norman’s estate.

Page 67: 1 McLAUGHLIN MULTISTATE BAR PREPARATION COURSE Class 5 Property I Questions 1-51 Copyright 2003 Walter H. McLaughlin

67

Part V Facts

• Two year ago, Louis married Zelda. They had no children.

• Translation

• In 1989 Louis marries Zelda. Louis has no children other than Norman who predeceased him.

• Analysis

• Zelda has a life estate, contingent on her surviving Louis and being his widow. The life estate does not violate the rule against perpetuities because it will vest, if at all, at the death of Louis who was a life in being in 1931 when the gift was created.

Page 68: 1 McLAUGHLIN MULTISTATE BAR PREPARATION COURSE Class 5 Property I Questions 1-51 Copyright 2003 Walter H. McLaughlin

68

• Part VI Facts• This year, Louis died testate, survived by his widow, Zelda,

to whom he left his entire estate. The common-law Rule Against Perpetuities is unchanged by statute in the jurisdiction.

• Translation• Louis dies in 1991, Zelda is his widow and sole legatee and

devisee.• Analysis.• Louis’s life estate terminates at his death.• Zelda’s life estate vests and becomes possessory at the

termination of Louis life estate, because the conditions precedent have been satisfied. She has survived Louis and is his widow. Zelda takes no interest in Greenacre from Louis’ estate because Louis only had a life estate.

Page 69: 1 McLAUGHLIN MULTISTATE BAR PREPARATION COURSE Class 5 Property I Questions 1-51 Copyright 2003 Walter H. McLaughlin

69

D Is Right• There are two issues in this question: whether Zelda’s life interest is

valid and whether the remainder interest in Norman is valid under the rule against perpetuities. Zelda’s life interest is valid because of part VI. She is Louis’ widow, and her interest vested immediately upon the death of Louis who was a life in being at the time his grandfather created the estate.

• The Red Cross has a valid vested remainder because of part IV. Norman as a child of Louis had a remainder interest which was contingent at the time it was created because Norman had not yet been born and he was unascertained at that time. However, it became a vested remainder as soon as he was born. since there was no condition precedent to Norman’s taking his interest. He did not have to survive either Louis or Louis’ widow to take. It had to vest during a life in being, namely, Norman’s father and is valid under the rule against perpetuities. A vested remainder is devisable and passed to the American Red Cross under Norman’s will.

Page 70: 1 McLAUGHLIN MULTISTATE BAR PREPARATION COURSE Class 5 Property I Questions 1-51 Copyright 2003 Walter H. McLaughlin

70

A, B and C Are Wrong• A Albert’s reversion created in I and devised in II terminates on the

birth of Norman. Even though Norman was unborn on the date of the conveyance, his interest vested upon his birth since there was no condition precedent to his taking his remainder interest after the death of Louis’s widow. Because he had predeceased Louis’ widow, his devisees would take the vested remainder at the death of Louis’ widow. Therefore the interest does not go back to Albert’s estate and then to First Church. Moreover, Zelda has a valid life interest in Greenacre because of the analysis in part VI.

• B Zelda has a valid life estate because of part VI, but no remainder interest because of part IV and part V. Even though Norman was unborn on the date of the conveyance, his interest vested upon his birth since there was no condition precedent to his taking his remainder interest after the death of Louis’s widow. Norman’s interest is a devisable vested remainder which went to the Red Cross rather than to his father Louis by intestacy.

• C While the Red Cross has a valid remainder interest because of IV and V; it is subject to a valid life interest in Zelda, Louis’ widow, because of part VI.

Page 71: 1 McLAUGHLIN MULTISTATE BAR PREPARATION COURSE Class 5 Property I Questions 1-51 Copyright 2003 Walter H. McLaughlin

71

Examsmanship Issue• This question is too long and complex for anyone

who is sensitive to the time issues on the multistate exam to properly analyze. It will take at least 5 minutes, the time allotted to three questions, to reach a reasonable choice. Therefore, when you see a question like this make a random guess and come back to it at the end if you have enough time.

Page 72: 1 McLAUGHLIN MULTISTATE BAR PREPARATION COURSE Class 5 Property I Questions 1-51 Copyright 2003 Walter H. McLaughlin

72

Governing Law• The Rule Against Perpetuities invalidates rights of

first refusal which might not be exercised within the period of the Rule.

Page 73: 1 McLAUGHLIN MULTISTATE BAR PREPARATION COURSE Class 5 Property I Questions 1-51 Copyright 2003 Walter H. McLaughlin

73

Question 15 Degree of Difficulty Hard• July 1998 Q 168• Three years ago Adam conveyed Blackacre to Betty for $50,000 by a

deed that provided: "By accepting this deed, Betty covenants for herself, her heirs and assigns, that the premises herein conveyed shall be used solely for residential purposes and, if the premises are used for nonresidential purposes, Adam, his heirs and assigns, shall have the right to repurchase the premises for the sum of one thousand dollars ($1,000)." In order to pay the $50,000 purchase price for Blackacre, Betty obtained a $35,000 mortgage loan from the bank. Adam had full knowledge of the mortgage transaction. The deed and mortgage were promptly and properly recorded in proper sequence. The mortgage, however, made no reference to the quoted language in the deed.

• Two years ago Betty converted her use of Blackacre from residential to commercial without the knowledge or consent of Adam or of the bank. Betty's commercial venture failed, and Betty defaulted on her mortgage payments to the bank. Blackacre now has a fair market value of $25,000.

Page 74: 1 McLAUGHLIN MULTISTATE BAR PREPARATION COURSE Class 5 Property I Questions 1-51 Copyright 2003 Walter H. McLaughlin

74

• The bank began appropriate foreclosure proceedings against Betty. Adam properly intervened, tendered $1,000, and sought judgment that Betty and the bank be ordered to convey Blackacre to Adam, free and clear of the mortgage.

• The common-law Rule Against Perpetuities is unmodified by statute.

• If the court rules against Adam, it will be because• (A) the provision quoted from the deed violates the Rule

Against Perpetuities.• (B) the Bank had no actual knowledge of, and did not

consent to, the violation of the covenant.• (C) the rights reserved by Adam were subordinated, by

necessary implication, to the rights of the bank as the lender of the purchase money.

• (D) the consideration of $1,000 was inadequate.

Page 75: 1 McLAUGHLIN MULTISTATE BAR PREPARATION COURSE Class 5 Property I Questions 1-51 Copyright 2003 Walter H. McLaughlin

75

D and C Are Wrong• D The consideration of $1,000 for the

reconveyance was part of a contract. Courts do not rewrite the contract between competent parties.

• C Bank’s interest in Blackacre as a mortgagee could not exceed the interest which Betty owned in the property and was subject to the same limitations to which Betty was subject. Unless Bank received a document directly from Adam waiving the covenant or agreeing to a period of time in which Bank could cure any breach, Adam’s right were not subordinated to Bank’s rights.

Page 76: 1 McLAUGHLIN MULTISTATE BAR PREPARATION COURSE Class 5 Property I Questions 1-51 Copyright 2003 Walter H. McLaughlin

76

B Is Wrong• Bank took a mortgage on Betty’s interest in

Blackacre. That interest was subject to the right to repurchase which was contained in the deed and recorded just before the mortgage. Bank thus had actual knowledge of the existence of the covenant. To protect its interest as mortgagee, Bank should have had a covenant in the mortgage that Betty could not convert the property to a business use, and should have inspected the property on a regular basis so that it could prevent Betty from converting. It then could have threatened to take possession for breach of a mortgage covenant if Betty started to convert.

Page 77: 1 McLAUGHLIN MULTISTATE BAR PREPARATION COURSE Class 5 Property I Questions 1-51 Copyright 2003 Walter H. McLaughlin

77

A Is Right• The right to repurchase property upon the happening

of an event is treated like a contingent remainder for the purpose of the rule against perpetuities, with the vesting occurring at the time that the event giving rise to the right to repurchase occurs. Here that right to repurchase bound not only Betty personally, but her heirs and assigns and was given not only to Adam, but also to his heirs and assigns. Therefore, the right to repurchase need not have vested within twenty-one years and lives in being and was invalid under the common law rule against perpetuities.

Page 78: 1 McLAUGHLIN MULTISTATE BAR PREPARATION COURSE Class 5 Property I Questions 1-51 Copyright 2003 Walter H. McLaughlin

78

Governing Law• A power of appointment permits the holder of the

power at either during that holder’s lifetime or in the holder’s will to direct the disposition of assets set up in a trust by the grantor of that power.

• A power of appointment is special if the potential beneficiaries of the exercise of that power are limited, and general if the holder of the power can direct the assets to anyone.

• Jurisdictions are split on the issue of whether the residuary clause of a will which does not specifically refer to a general power of appointment in fact exercises that power in favor of the residuary beneficiary.

Page 79: 1 McLAUGHLIN MULTISTATE BAR PREPARATION COURSE Class 5 Property I Questions 1-51 Copyright 2003 Walter H. McLaughlin

79

Question 16 Degree of Difficulty Hard• July 1998 Q 48• Arthur s estate plan included a revocable trust established 35 years

ago with ABC Bank as trustee. The principal asset of the trust has always been Blackacre, a very profitable, debt-free office building. The trust instrument instructs the trustee to pay the net income to Arthur for life, and, after the death of Arthur, to pay the net income to his wife, Alice, for life; and, after her death, to distribute the net trust estate as she may appoint by will, or in default of her exercise of this power of appointment, to my son (her stepson), Charles.

• Arthur died 30 years ago survived by Alice and Charles. Arthur had not revoked or amended the trust agreement. A few years after Arthur’s death, Alice remarried; she then had a child, Marie; was widowed for a second time; and, last year, died. Her will contained only one dispositive provision: I give my entire estate to my daughter, Marie, and I intentionally make no provision for my stepson, Charles.

Page 80: 1 McLAUGHLIN MULTISTATE BAR PREPARATION COURSE Class 5 Property I Questions 1-51 Copyright 2003 Walter H. McLaughlin

80

• Marie is now 22 years old. The common-law Rule Against Perpetuities is unmodified by statute in the jurisdiction. There are no other applicable statutes.

• Charles brought an appropriate action against Marie to determine who was entitled to the net trust estate and thus to Blackacre.

• If the court rules for Marie, it will be because• (A) Alice s life estate and general power of appointment

merge into complete ownership in Alice. • (B) the Rule Against Perpetuities does not apply to

general powers of appointment. • (C) the jurisdiction deems the entire estate to be a

reference to Blackacre or to Alice’s general power of appointment.

• (D) Alice intended that Charles should not benefit by reason of her death.

Page 81: 1 McLAUGHLIN MULTISTATE BAR PREPARATION COURSE Class 5 Property I Questions 1-51 Copyright 2003 Walter H. McLaughlin

81

B Is Wrong• The rule against perpetuities does apply to the

exercise of general powers of appointment but, when determining validity, lives in being at the time of the exercise of the power are considered measuring lives even though they were not lives in being at the time of the creation of the power. The rule against perpetuities is satisfied in this instance without reference to the rule concerning powers of appointment because Alice is a life in being at the creation of the interest. The interest vests immediately on Alice’s death whether that interest goes to her daughter, Marie, or to her stepson, Charles.

Page 82: 1 McLAUGHLIN MULTISTATE BAR PREPARATION COURSE Class 5 Property I Questions 1-51 Copyright 2003 Walter H. McLaughlin

82

A and D Are Wrong• A Alice’s life estate and her power of appointment over the

remainder are distinct property interests and did not merge into complete ownership for Alice. Alice did not have the power to invade the principal of Arthur’s trust during her lifetime. Her sole right was to determine who was to receive that principal at her death through her general power of appointment.

• D It is clear that Alice intended that the remainder interest in the trust go to Marie. However, it will not be distributed to Marie unless Alice exercised her power of appointment by will. Therefore, the critical issue is whether she in fact exercised it by the language of her will.

Page 83: 1 McLAUGHLIN MULTISTATE BAR PREPARATION COURSE Class 5 Property I Questions 1-51 Copyright 2003 Walter H. McLaughlin

83

C Is Right• The critical issue to be determined is whether Alice

exercised her power of appointment in favor of her daughter, Marie. She could have removed any doubt on this issue by explicitly exercising that power in her will. That power will have been exercised by Alice’s will if the law in the jurisdiction provides that a general power of appointment is exercised by the language in Alice’s will which gives her “entire estate” to Marie and specifically excludes Charles.

Page 84: 1 McLAUGHLIN MULTISTATE BAR PREPARATION COURSE Class 5 Property I Questions 1-51 Copyright 2003 Walter H. McLaughlin

84

Governing Law Restraint on Alienation

• A prohibition of a grantee's right to alienate property or a provision forfeiting an interest if the grantee attempts to alienate is invalid.

Page 85: 1 McLAUGHLIN MULTISTATE BAR PREPARATION COURSE Class 5 Property I Questions 1-51 Copyright 2003 Walter H. McLaughlin

85

Question 17 Degree of Difficulty Medium• By her validly executed will, Sallie devised a certain tract of

land to her son, Ben, for his life with remainder to such of Ben's children as should be living at his death, “Provided, however, that no such child of Ben shall mortgage or sell, or attempt to mortgage or sell, his or her interest in the property prior to attaining 25 years of age; and, if any such child of Ben shall violate this provision, then upon such violation, his or her interest shall pass to and become the property of the remaining children of Ben then living, share and share alike."

• Sallie's will included an identical provision for each of her four other children concerning four other tracts of land. The residuary clause of the will gave the residuary estate to Sallie's five children equally. Sallie died and was survived by the five children named in her will and by eleven grandchildren. Several additional grandchildren have since been born.

Page 86: 1 McLAUGHLIN MULTISTATE BAR PREPARATION COURSE Class 5 Property I Questions 1-51 Copyright 2003 Walter H. McLaughlin

86

• In an action for a declaration of rights, it was claimed that the attempted gifts to Sallie's grandchildren were entirely void and that the interests following the life estates to Sallie's children passed to the children absolutely by the residuary clause.

• Assuming that the action was properly brought with all necessary parties and with a guardian ad litem appointed to represent the interests of unborn and infant grandchildren, the decision should be that

• (A) the attempted gifts to grandchildren are void under the Rule Against Perpetuities.

• (B) the attempted gifts to grandchildren are void as unlawful restraints on alienation.

• (C) the provisions concerning grandchildren are valid and will be upheld according to their terms.

• (D) even if the provisions against sale or mortgage by the grandchildren are void, the remainders to grandchildren are otherwise valid and will be given effect.

Page 87: 1 McLAUGHLIN MULTISTATE BAR PREPARATION COURSE Class 5 Property I Questions 1-51 Copyright 2003 Walter H. McLaughlin

87

A, B and C Are Wrong; D Is Right• A Is the gift to the grandchildren valid under the rule against

perpetuities? The gift to the grandchildren does not violate the Rule Against Perpetuities because all of Sallie's children are lives in being at the beginning of the perpetuities period, her death; or they have already died, and the children of that child take immediately at her death. All of the grandchildren must be born during the lives of Sallie's children, and their interest vests at the death of Sallie's children, the lives in being.

• B The invalidity of the restraint on alienation does not make the gift to the grandchildren void. It merely makes it possible for them to alienate their interest without penalty before they reach age 25.

• C The will clause provides that if a grandchild attempts to alienate his interest before reaching 25, then his interest shall be forfeited and pass to his brothers and sisters. Such a forfeiture restraint, even for a limited time on a legal (as opposed to an equitable) remainder interest, is void.

• D The restraint on alienation is probably void but the gift is valid despite the restraint. The Rule Against Perpetuities does not invalidate the interests of the grandchildren.

Page 88: 1 McLAUGHLIN MULTISTATE BAR PREPARATION COURSE Class 5 Property I Questions 1-51 Copyright 2003 Walter H. McLaughlin

88

Governing Law Adverse Possession

• The sole possession by one co-tenant for the statutory period of adverse possession does not establish title in that co-tenant by adverse possession unless the tenant in possession engages in conduct which amounts to an ouster of his co-tenant by barring that co-tenant from the property for the statutory period.

Page 89: 1 McLAUGHLIN MULTISTATE BAR PREPARATION COURSE Class 5 Property I Questions 1-51 Copyright 2003 Walter H. McLaughlin

89

Question 18 Degree of Difficulty Hard• Arthur and Celia, brother and sister, both of legal age, inherited

Goodacre, their childhood home, from their father. They thereby became tenants in common.

• Goodacre had never been used as anything except a residence. Arthur had been residing on Goodacre with his father at the time his father died. Celia had been residing in a distant city. After their father's funeral, Arthur continued to live on Goodacre, but Celia returned to her own residence.

• There was no discussion between Arthur and Celia concerning their common ownership, nor had there ever been any administration of their father's estate. Arthur paid all taxes, insurance, and other carrying charges on Goodacre. He paid no rent or other compensation to Celia, nor did Celia request any such payment.

• Thirty years later, a series of disputes arose between Arthur and Celia for the first time concerning their respective rights to Goodacre. The jurisdiction where the land is located recognizes the usual common-law types of cotenancies, and there is no applicable legislation on the subject.

Page 90: 1 McLAUGHLIN MULTISTATE BAR PREPARATION COURSE Class 5 Property I Questions 1-51 Copyright 2003 Walter H. McLaughlin

90

• If Arthur claims the entire title to Goodacre in fee simple and brings an action against Celia to quiet title in himself, and if the state where the land is located has an ordinary 20-year adverse possession statute, the decision should be for

• (A) Arthur, because during the past 30 years Arthur has exercised the type of occupancy ordinarily considered sufficient to satisfy the adverse possession

requirements. • (B) Arthur, because the acts of the parties indicate Celia's

intention to renounce her right to inheritance. • (C) Celia, because there is no evidence that Arthur has

performed sufficient acts to constitute her ouster. • (D) Celia, because one co-tenant cannot acquire title by

adverse possession against another.

Page 91: 1 McLAUGHLIN MULTISTATE BAR PREPARATION COURSE Class 5 Property I Questions 1-51 Copyright 2003 Walter H. McLaughlin

91

D, A and B Are Wrong; C Is Right• D A co-tenant can obtain title by adverse possession if he ousts his

cotenant and remains in exclusive possession for twenty years thereafter.

• A This choice correctly describes the standard for adverse possession when a co-tenancy doesn’t exist.

• B By permitting Arthur to enjoy exclusive possession, co-tenant Celia did not renounce her interest in the property and did not indicate acquiescence to adverse possession.

• C If the use is permissive, one co-tenant's exclusive use of jointly held property is not sufficient to obtain title by possession, as long as that use is permissive. One co-tenant's possession of joint property becomes adverse only when he ousts his co-tenant. In order to oust a co-tenant, the tenant has to physically eject or bar the co-tenant from possession, or explicitly announce the intention to do so to the excluded co-tenant. Arthur did not Oust Celia.

Page 92: 1 McLAUGHLIN MULTISTATE BAR PREPARATION COURSE Class 5 Property I Questions 1-51 Copyright 2003 Walter H. McLaughlin

92

Governing Law Joint Tenancy

• When parties hold property in either a joint tenancy or in a tenancy by the entirety, at the death of one of the tenants complete title vests in the surviving tenant and an involuntary lien placed against the interest of the dead cotentant does not burden the surviving cotentant.

Page 93: 1 McLAUGHLIN MULTISTATE BAR PREPARATION COURSE Class 5 Property I Questions 1-51 Copyright 2003 Walter H. McLaughlin

93

Question 19 Degree of Difficulty Medium• The following facts concern a tract of land in a state which

follows general United States law. Each instrument is in proper form, and recorded, marital property rights were waived when necessary, and each person named was adult and competent at the time of the named transaction.

• 1. In 1940 Oleg, the owner, conveyed his interest in fee simple "to my brothers, Bob and Bill, their heirs and assigns as joint tenants with right of survivorship”.

• 2. In 1950 Bob died, devising his interest to his only child, “Charles, for life, and then to Charles' son, Sam, for life, and then to Sam’s children, their heirs and assigns.”

• 3. In 1970 Bill died, devising his interest “to my friend, Frank, his heirs and assigns.”

• 4. In 1972 Frank conveyed by quitclaim deed “to Paul, his heirs and assigns whatever right, title and interest I own.”

Page 94: 1 McLAUGHLIN MULTISTATE BAR PREPARATION COURSE Class 5 Property I Questions 1-51 Copyright 2003 Walter H. McLaughlin

94

• Paul has never married. Paul has contracted to convey marketable record title in the land to Patrick. Can Paul do so?

• (A) Yes, without joinder of any other person in the conveyance.

• (B) Yes, if Charles, Sam, and Sam's only child (Gene, aged 25) will join in the conveyance.

• (C) No, regardless of who joins in the conveyance, because Sam may have additional children

whose interests cannot be defeated.

• (D) No, regardless of who joins in the conveyance, because a title acquired by quitclaim deed is impliedly unmerchantable.

Page 95: 1 McLAUGHLIN MULTISTATE BAR PREPARATION COURSE Class 5 Property I Questions 1-51 Copyright 2003 Walter H. McLaughlin

95

B C and D Are Wrong; A Is Right• B C A The answer to this question depends upon the

disposition of Bob's interest in the property at his death. Since Bob and Bill held the property as joint tenants and nothing was done to sever the joint tenancy before Bob's death, Bob's interest passed to Bill by right of survivorship at Bob's death, and his heirs and devisees have no interest in the property. Therefore, the provisions of Bob's will are irrelevant, and the entire interest in the property passed at Bill's death to Frank and, by Frank's deed, to Paul. Therefore, Paul can convey without any other person.

• D A quitclaim deed conveys all of the title which a grantor has in the property conveyed. If the grantor has marketable title, the grantee acquires marketable title from a quitclaim deed.

Page 96: 1 McLAUGHLIN MULTISTATE BAR PREPARATION COURSE Class 5 Property I Questions 1-51 Copyright 2003 Walter H. McLaughlin

96

Question 20 Degree of Difficulty Medium• July 1991 Q 139 • Homer and Ethel were jointly in possession of Greenacre in fee

simple as tenants in common. They joined in a mortgage of Greenacre to Fortunoff Bank. Homer erected a fence along what he considered to be the true boundary between Greenacre and the adjoining property, owned by Mitchell.

• Shortly thereafter, Homer had an argument with Ethel and gave up his possession to Greenacre. The debt secured by the mortgage had not been paid.

• Mitchell surveyed his land and found that the fence erected a year earlier by Homer did not follow the true boundary. Part of the fence was within Greenacre. Part of the fence encroached on Mitchell's land. Mitchell and Ethel executed an agreement fixing the boundary line in accordance with the fence constructed by Homer. The agreement, which met all the formalities required in the jurisdiction, was promptly and properly recorded.

Page 97: 1 McLAUGHLIN MULTISTATE BAR PREPARATION COURSE Class 5 Property I Questions 1-51 Copyright 2003 Walter H. McLaughlin

97

• A year after the agreement was recorded, Homer temporarily reconciled his differences with Ethel and resumed joint possession of Greenacre. Thereafter, Homer repudiated the boundary line agreement and brought an appropriate action against Mitchell and Ethel to quiet title along the original true boundary.

• In such action, Homer will• (A) win, because Fortunoff Bank was not a party tothe

agreement. • (B) win, because one tenant in common cannot bind

another tenant in common to a boundary-line agreement. • (C) lose, because the agreement, as a matter of law, was

mutually beneficial to Ethel and Homer.• (D) lose, because Ethel was in sole possession of said

premises at the time the agreement was signed.

Page 98: 1 McLAUGHLIN MULTISTATE BAR PREPARATION COURSE Class 5 Property I Questions 1-51 Copyright 2003 Walter H. McLaughlin

98

A, C and D Are Wrong; B Is Right• A Homer will prevail because he did not sign the

agreement. He does not need to use the lack of consent of the bank to invalidate it.

• C The beneficial nature of the agreement to Homer is irrelevant. The essential point is that Homer was an owner of one half of Greenacre and did not sign the agreement.

• D Even though Ethel was in sole possession of Greenacre, she was not the sole owner of Greenacre. Homer never signed the agreement and is not bound by it.

• B Homer was still a one-half owner of the property as a tenant in common at the time that Ethel executed the agreement with Mitchell. Since he did not sign it, the agreement is not binding on him. Ethel’s signature does not bind Homer.

Page 99: 1 McLAUGHLIN MULTISTATE BAR PREPARATION COURSE Class 5 Property I Questions 1-51 Copyright 2003 Walter H. McLaughlin

99

Governing Law Joint Tenancy

• The granting of a mortgage by one joint tenant does not transform the tenancy into a tenancy in common in a state adopting the lien theory of mortgages, but it does in a title-theory state.

Page 100: 1 McLAUGHLIN MULTISTATE BAR PREPARATION COURSE Class 5 Property I Questions 1-51 Copyright 2003 Walter H. McLaughlin

100

Question 21 Degree of Difficulty Medium• Johnson and Tenniel owned Brownacre as joint tenants with

the right of survivorship. Johnson executed a mortgage on Brownacre to Lowden in order to secure a loan. Subsequently, but before the indebtedness was paid to Lowden, Johnson died intestate with Stokes as her only heir at law. The jurisdiction at which Brownacre is located recognizes the title theory of mortgages.

• In an appropriate action, the court should determine that title to Brownacre is vested

• (A) in Tenniel, with the entire interest subject to the mortgage.

• (B) in Tenniel, free and clear of the mortgage. • (C) half in Tenniel, free of the mortgage, and half in

Stokes, subject to the mortgage. • (D) half in Tenniel and half in Stokes, with both subject

to the mortgage.

Page 101: 1 McLAUGHLIN MULTISTATE BAR PREPARATION COURSE Class 5 Property I Questions 1-51 Copyright 2003 Walter H. McLaughlin

101

A, D and B Are Wrong• A D Tenniel's interest can never be subject to the

mortgage, since he didn't join in it. A is also wrong because Tenniel's right of survivorship cannot survive Johnson's mortgage of his interest.

• B would be the correct answer only in a lien theory state that held that one tenant's mortgage does not destroy a joint tenancy. In such a state, the mortgagee's interest in the property would be extinguished with Johnson's death, since his death would also terminate Johnson's estate.

Page 102: 1 McLAUGHLIN MULTISTATE BAR PREPARATION COURSE Class 5 Property I Questions 1-51 Copyright 2003 Walter H. McLaughlin

102

C Is Right• This problem involves a joint tenancy where one joint

tenant mortgages his interest in the property. Of course, the mortgage only attaches to the interest of the tenant who executes the mortgage; the interests and rights of other cotenants cannot be impinged by the actions of one cotenant. But, more importantly, the majority rule (adopted by all title theory states) is that a mortgage by one joint tenant converts the joint tenancy into a tenancy in common (with no right of survivorship). Thus, in this case, Johnson's execution of a mortgage converted the tenancy into a tenancy in common, so that when Johnson died, his half interest passed to his heir, Stokes, subject to the mortgage.

Page 103: 1 McLAUGHLIN MULTISTATE BAR PREPARATION COURSE Class 5 Property I Questions 1-51 Copyright 2003 Walter H. McLaughlin

103

Governing Law Joint Tenancies and Tenancies By the Entirety

• Both a joint tenancy and a tenancy by the entirety vest complete title in the surviving tenant if they are in operation at the death of one of the tenants.

• A joint tenancy is severed by the alienation of one tenant’s interest. It then becomes a tenancy in common with no survivorship rights between the grantee of the tenant who conveyed and the remaining original tenant.

• A tenancy by the entirety can only be created between husband and wife who are actually married to each other. Neither party can terminate the tenancy by the entirety by a unilateral act during the marriage. An attempt to do so vests no property interest in the grantee.

Page 104: 1 McLAUGHLIN MULTISTATE BAR PREPARATION COURSE Class 5 Property I Questions 1-51 Copyright 2003 Walter H. McLaughlin

104

Question 22 Degree of Difficulty Hard• Lawnacre was conveyed to Celeste and Donald by a deed

which, in the jurisdiction in which Lawnacre is situated, created a cotenancy in equal shares and with the right of survivorship. The jurisdiction has no statute directly applicable to any of the problems posed.

• Celeste, by deed, conveyed "my undivided one-half interest in Lawnacre” to Paul. Celeste has since died.

• In an appropriate action between Paul and Donald in which title to Lawnacre is at issue, Donald will

• (A) prevail, because he is the sole owner of Lawnacre. • (B) prevail if, but only if, the cotenancy created in

Celeste and Donald was a tenancy by the entirety. • (C) not prevail if he had knowledge of the

conveyance prior to Celeste's death. • (D) not prevail, because Paul and Donald own Lawnacre

as tenants in common.

Page 105: 1 McLAUGHLIN MULTISTATE BAR PREPARATION COURSE Class 5 Property I Questions 1-51 Copyright 2003 Walter H. McLaughlin

105

A and D Are Wrong• A Donald is the sole owner of Lawnacre only if the

interest created was one of the two possibilities, a tenancy by the entirety. He only owns an undivided one half interest if the form of co-tenancy in equal shares was a joint tenancy.

• D Paul and Donald own Lawnacre in equal shares only if the interest created was a joint tenancy. Donald is the sole owner if the form of cotenancy in equal shares was a tenancy by the entirety.

Page 106: 1 McLAUGHLIN MULTISTATE BAR PREPARATION COURSE Class 5 Property I Questions 1-51 Copyright 2003 Walter H. McLaughlin

106

B Is Right and C Is Wrong• B It correctly limits the situation in which Donald will own

all of Lawnacre to the case where Donald and Celeste held as tenants by the entirety because the conveyance by Celeste is invalid under those circumstances.

• C The type of interest held rather than knowledge of the conveyance by the other cotenant is controlling in determining if the right of survivorship has been terminated. If the interest was a tenancy by the entirety, the right of survivorship would not have been terminated, even if Donald knew of Celeste's deed to Paul. On the other hand, if the interest were a joint tenancy, the right of survivorship would have been terminated even if Donald were ignorant of Celeste's deed.

Page 107: 1 McLAUGHLIN MULTISTATE BAR PREPARATION COURSE Class 5 Property I Questions 1-51 Copyright 2003 Walter H. McLaughlin

107

Examsmanship Note• The facts only state that Celeste and Donald owned a

cotenancy in equal shares with a right of survivorship. That cotenancy could either be a joint tenancy or, if Celeste and Donald were married, it could be a tenancy by the entirety. If it were a joint tenancy, the conveyance from Celeste to Paul would have severed the joint tenancy, creating a tenancy in common between Paul and Donald. On the other hand, if Celeste and Donald held as tenants by the entirety, the unilateral conveyance by Celeste to Paul would be a nullity, and Donald would inherit Celeste's interest by right of survivorship. A and D are tempting choices, because they are each right under one possible scenario. However, they are wrong because another choice correctly incorporates both rules.

Page 108: 1 McLAUGHLIN MULTISTATE BAR PREPARATION COURSE Class 5 Property I Questions 1-51 Copyright 2003 Walter H. McLaughlin

108

Governing Law• Open, notorious and exclusive possession by one

cotenant for the statutory period will not establish adverse possession unless the other cotenant was ousted at the beginning of that period.

• When property is partitioned, the decree contains an accounting between cotenants concerning payments of expenses and collection of rents.

Page 109: 1 McLAUGHLIN MULTISTATE BAR PREPARATION COURSE Class 5 Property I Questions 1-51 Copyright 2003 Walter H. McLaughlin

109

Question 23 Degree of Difficulty Medium• July 1998 Q 25• Alex and Betty, who were cousins, acquired title in fee

simple to Blackacre, as equal tenants in common, by inheritance from Angela, their aunt. During the last 15 years of her lifetime, Angela allowed Alex to occupy an apartment in the house on Blackacre, to rent the other apartment in the house to various tenants, and to retain the rent. Alex made no payments to Angela; and since Angela’s death 7 years ago, he has made no payments to Betty.

• For those 22 years, Alex has paid the real estate taxes on Blackacre, kept the building on Blackacre insured, and maintained the building.

• At all times, Betty has lived in a distant city and has never had anything to do with Angela, Alex, or Blackacre.

Page 110: 1 McLAUGHLIN MULTISTATE BAR PREPARATION COURSE Class 5 Property I Questions 1-51 Copyright 2003 Walter H. McLaughlin

110

• Recently, Betty needed money for the operation of her business and demanded that Alex join her in selling Blackacre. Alex refused.

• The period of time to acquire title by adverse possession in the jurisdiction is 10 years. There is no other applicable statute.

• Betty brought an appropriate action against Alex for partition. Alex asserted all available defenses and counterclaims.

• In that action, the court should• (A) deny partition and find that title has vested in Alex by adverse

possession. • (B) deny partition, confirm the tenancy in common, but require an

accounting to determine if either Betty or Alex is indebted to the other on account of the rental payment, taxes, insurance premiums, and maintenance costs.

• (C) grant partition and require, as an adjustment, an accounting to determine if either Betty or Alex is indebted to the other on account of the rental payments, taxes, insurance premiums, and maintenance costs.

• (D) grant partition to Betty and Alex as equal owners, but without an accounting.

Page 111: 1 McLAUGHLIN MULTISTATE BAR PREPARATION COURSE Class 5 Property I Questions 1-51 Copyright 2003 Walter H. McLaughlin

111

A and B Are Wrong• A Alex does not have title by adverse possession for two

reasons. First, his occupancy of the premises during the lifetime of Angela was permissive. At best, his occupancy was adverse only during the seven years since Angela’s death, three years short of the ten-year period required. Secondly, he became a tenant-in-common with Betty after Angela’s death. His conduct since that time does not constitute an ouster of Betty, the conduct required to establish the commencement of the period of adverse possession against a co-tenant.

• B Except where specifically prohibited by statute or where there is a tenancy by the entirety, co-tenants, in this case Alex and Betty, have an absolute right to partition.

Page 112: 1 McLAUGHLIN MULTISTATE BAR PREPARATION COURSE Class 5 Property I Questions 1-51 Copyright 2003 Walter H. McLaughlin

112

D Is Wrong and C Is Right• Alex has occupied the property, collected all of the

rents and paid all of the expenses since Angela’s death. While he owes no rent on the apartment he occupied, a partition proceeding must adjust the interests of the co-owners based upon their actions with respect to the property since they became co-owners.

Page 113: 1 McLAUGHLIN MULTISTATE BAR PREPARATION COURSE Class 5 Property I Questions 1-51 Copyright 2003 Walter H. McLaughlin

113

Governing Law Tenants In Common• There is no right of survivorship when parties own

property as tenants in common.

• One tenant in common owes the other tenant a fiduciary duty to maintain his proportionate ownership of the property by paying his proportionate share of the debt if one cotenant acquires the property at a foreclosure sale.

Page 114: 1 McLAUGHLIN MULTISTATE BAR PREPARATION COURSE Class 5 Property I Questions 1-51 Copyright 2003 Walter H. McLaughlin

114

Question 24 Degree of Difficulty Hard• July 1998 Q 131• Alpha and Beta owned Greenacre, a large farm, in fee simple as

tenants in common, each owning an undivided one-half interest. For five years Alpha occupied Greenacre and conducted farming operations. Alpha never accounted to Beta for any income, but Alpha did pay all real estate taxes when the taxes were due and kept the buildings located on Greenacre insured against loss from fire, storm, and flood. Beta lived in a distant city and was interested only in realizing a profit from the sale of the land when market conditions produced the price Beta wanted.

• Alpha died intestate survived by Hera, Alpha's sole heir. Thereafter, Hera occupied Greenacre but was inexperienced in farming operations. The result was a financial disaster. Hera failed to pay real estate taxes for two years. The appropriate governmental authority held a tax sale to recover the taxes due. At such sale Beta was the only bidder and obtained a conveyance from the appropriate governmental authority upon payment of an amount sufficient to discharge the amounts due for taxes, plus interest and penalties, and the costs of holding the tax sale. The amount paid was one-third of the reasonable market value of Greenacre.

Page 115: 1 McLAUGHLIN MULTISTATE BAR PREPARATION COURSE Class 5 Property I Questions 1-51 Copyright 2003 Walter H. McLaughlin

115

• Thereafter, Beta instituted an appropriate action against Hera to quiet title in and to recover possession of Greenacre. Hera asserted all defenses available to Hera.

• Except for the statutes related to real estate taxes and tax sales, there is no applicable statute.

• In this lawsuit, Beta is entitled to a decree quieting title so that Beta is the sole owner in fee simple of Greenacre

• (A) because Beta survived Alpha. • (B) because Hera defaulted in the obligations undertaken

by Alpha.• (C) unless Hera pays Beta one-half of the reasonable

market value of Greenacre. • (D) unless Hera pays Beta one-half of the amount Beta

paid for the tax deed.

Page 116: 1 McLAUGHLIN MULTISTATE BAR PREPARATION COURSE Class 5 Property I Questions 1-51 Copyright 2003 Walter H. McLaughlin

116

A and B Are Wrong• A Because Alpha and Beta owned Greenacre as

tenants in common and not in joint tenancy, Beta did not take Alpha’s share of Greenacre on his death. Title to Alpha’s undivided one-half interest vested in Hera, his sole heir.

• B Alpha as a tenant in common voluntarily paid expenses while he used the property. When Hera inherited Alpha’s undivided one-half interest, he only had the usual obligations of a tenant in common to pay expenses, not special obligations undertaken by Alpha. Moreover, even if he did default on them, this default by itself would not cause him to lose title.

Page 117: 1 McLAUGHLIN MULTISTATE BAR PREPARATION COURSE Class 5 Property I Questions 1-51 Copyright 2003 Walter H. McLaughlin

117

D Is Right and C Is Wrong• One tenant in common owes a fiduciary duty to the

other tenant in common when he is the successful bidder at a mortgagee foreclosure sale or tax sale. When he purchased the property at the tax foreclosure sale, Beta was required to offer Hera a one-half interest in the property if Hera paid one half of the taxes, not if he pays one half of the value of the property. If Hera fails to pay his share of the taxes, then Beta is the sole owner of Blackacre.

Page 118: 1 McLAUGHLIN MULTISTATE BAR PREPARATION COURSE Class 5 Property I Questions 1-51 Copyright 2003 Walter H. McLaughlin

118

Governing Law Lateral Support• The owner of land which is in its natural state can

successfully sue an adjoining landowner who disturbs that support on a strict liability theory without showing negligence.

• The owner of improved land can successfully sue an adjoining landowner who disturbs that support only if he proves that the adjoining landowner is negligent.

Page 119: 1 McLAUGHLIN MULTISTATE BAR PREPARATION COURSE Class 5 Property I Questions 1-51 Copyright 2003 Walter H. McLaughlin

119

Question 25 Degree of Difficulty Easy• July 1998 Q 87• Bill owned in fee simple Lot 1 in a properly approved subdivision,

designed and zoned for industrial use. Gail owned the adjoining Lot 2 in the same subdivision. The plat of the subdivision was recorded as authorized by statute.

• Twelve years ago, Bill erected an industrial building wholly situated on Lot 1 but with one wall along the boundary common with Lot 2. The construction was done as authorized by a building permit, validly obtained under applicable statutes, ordinances, and regulations. Further, the construction was regularly inspected and passed as being in compliance with all building code requirements.

• Lot 2 remained vacant until six months ago, when Gail began excavation pursuant to a building permit authorizing the erection of an industrial building situated on Lot 2 but with one wall along the boundary common with Lot 1. The excavation caused subsidence of a portion of Lot 1 that resulted in injury to Bill s building. The excavation was not done negligently or with any malicious intent to injure. In the jurisdiction, the time to acquire title by adverse possession or rights by prescription is 10 years.

Page 120: 1 McLAUGHLIN MULTISTATE BAR PREPARATION COURSE Class 5 Property I Questions 1-51 Copyright 2003 Walter H. McLaughlin

120

• Bill brought an appropriate action against Gail to recover damages resulting from the injuries to the building on Lot 1.

• In such lawsuit, judgment should be for• (A) Bill, if, but only if, the subsidence would have

occurred without the weight of the building on Lot 1. • (B) Bill, because a right for support, appurtenant to Lot 1,

had been acquired by adverse possession or prescription. • (C) Gail, because Lots 1 and 2 are urban land, as

distinguished from rural land and, therefore, under the circumstances Bill had the duty to protect any improvements on Lot 1.

• (D) Gail, because the construction and the use to be made of the building, were both authorized by the applicable law.

Page 121: 1 McLAUGHLIN MULTISTATE BAR PREPARATION COURSE Class 5 Property I Questions 1-51 Copyright 2003 Walter H. McLaughlin

121

C, D and B Are Wrong; A Is Right• C The duty to provide lateral support applies to both urban and

rural land. The duty is to avoid negligence with respect to improved land, and is absolute with respect to land in its natural state.

• D The fact that the zoning and building codes permit a building on lot 2 do not insulate the lot owner from liability if his actions are negligent and cause subsidence on his neighbor’s building or even without negligence are such that they have would caused subsidence to his neighbor’s land if it were in its natural state.

• B There has been no adverse possession because Bill’s building was on his own land and did not encroach on Gail’s land. The law of lateral support has developed independently of any theory of adverse possession.

• A The facts indicate that Gail’s excavation was not negligent with respect to Gail’s obligation to provide lateral support. Therefore, there is no cause of action for the subsidence of Bill’s building unless Bill is entitled to proceed under the strict liability standard which applies only when he can prove that Gail’s excavation would have caused the subsidence of his land even if it were in its natural state.

Page 122: 1 McLAUGHLIN MULTISTATE BAR PREPARATION COURSE Class 5 Property I Questions 1-51 Copyright 2003 Walter H. McLaughlin

122

Governing Law• Nuisance requires activity on a parcel of land which

unreasonably interferes with the use and enjoyment of the plaintiff’s land.

Page 123: 1 McLAUGHLIN MULTISTATE BAR PREPARATION COURSE Class 5 Property I Questions 1-51 Copyright 2003 Walter H. McLaughlin

123

Question 26 Degree of Difficulty Medium• July 1998 Q 175• Adam owns Townacres in fee simple, and Bess owns the

adjoining Greenacres in fee simple. Adam has kept the lawns and trees on Townacres trimmed and neat. Bess "lets nature take its course" at Greenacres. The result on Greenacres is a tangle of underbrush, fallen trees, and standing trees that are in danger of losing limbs. Many of the trees on Greenacres are near Townacres. In the past, debris and large limbs have been blown from Greenacres onto Townacres. By local standards Greenacres is an eyesore that depresses market values of real property in the vicinity, but the condition of Greenacres violates no applicable laws or ordinances.

Page 124: 1 McLAUGHLIN MULTISTATE BAR PREPARATION COURSE Class 5 Property I Questions 1-51 Copyright 2003 Walter H. McLaughlin

124

• Adam demanded that Bess keep the trees near Townacres trimmed. Bess refused.

• Adam brought an appropriate action against Bess to require Bess to abate what Adam alleges to be a nuisance. In the lawsuit, the only issue is whether the condition of Greenacres constitutes a nuisance.

• The strongest argument that Adam can present is that the condition of Greenacres

• (A) has an adverse impact on real estate values. • (B) poses a danger to the occupants of Townacres. • (C) violates community aesthetic standards. • (D) cannot otherwise be challenged under any law

or ordinance.

Page 125: 1 McLAUGHLIN MULTISTATE BAR PREPARATION COURSE Class 5 Property I Questions 1-51 Copyright 2003 Walter H. McLaughlin

125

C, D and A Are Wrong; B Is Right• C Violation of aesthetic standards does not provide the

basis for an action in nuisance because such a violation does not interfere with the use and enjoyment of the neighboring land.

• D The fact that there is no cause of action under a zoning ordinance does not give rise to a cause of action in nuisance.

• A Nuisance requires an activity which interferes with the use and enjoyment of neighboring property, not one which only interferes with its value.

• B Nuisance requires activity on a parcel of land which unreasonably interferes with the use and enjoyment of the plaintiff’s land. If Beth’s failure to maintain her property results in debris and trees to remain in a dangerous condition where they can blow onto the adjoining Townacres property and injure its occupants, then Adam can maintain an action in nuisance.

Page 126: 1 McLAUGHLIN MULTISTATE BAR PREPARATION COURSE Class 5 Property I Questions 1-51 Copyright 2003 Walter H. McLaughlin

126

Governing Law Appurtenant Easements• An easement is appurtenant if it burdens one parcel

of land , the servient estate, for the specific benefit on another parcel, the dominant estate.

• The transfer of title to the dominant estate transfers any easement rights held by the owner of the dominant estate even if the easement is not specifically transferred in the deed.

• An easement by implication or necessity arises only when there is no express easement.

Page 127: 1 McLAUGHLIN MULTISTATE BAR PREPARATION COURSE Class 5 Property I Questions 1-51 Copyright 2003 Walter H. McLaughlin

127

Question 27 Degree of Difficulty Easy• July 1991 Q 55• Olwen owned 80 acres of land, fronting on a town road. Two years

ago Olwen sold to Buck the back 40 acres. The 40 acres sold to Buck did not adjoin any public road. Olwen's deed to Buck expressly granted a right-of-way over a specified strip of Olwen's retained 40 acres, so Buck could reach the town road. The deed was promptly and properly recorded.

• Last year, Buck conveyed the back 40 acres to Sam. They had discussed the right-of-way over Olwen's land to the road, but Buck's deed to Sam made no mention of it. Sam began to use the right-of-way as Buck had, but Olwen sued to enjoin such use by Sam.

• The court should decide for• (A) Sam, because he has an easement by implication. • (B) Sam, because the easement appurtenant passed to him as a

result of Buck's deed to him. • (C) Olwen, because Buck's easement in gross was not transferable.• (D) Olwen, because Buck's deed failed expressly to transfer the

right-of-way to Sam.

Page 128: 1 McLAUGHLIN MULTISTATE BAR PREPARATION COURSE Class 5 Property I Questions 1-51 Copyright 2003 Walter H. McLaughlin

128

A, C and D Are Wrong; B Is Right • A The easement is express because it was created by a valid written

conveyance which was recorded. An easement by implication cannot arise when an express easement satisfies the rights of access of the landlocked parcel.

• C The easement is appurtenant to the back land because it was designed to provide access to that land which is the dominant estate. An easement in gross such as a utility easement benefits no specific land.

• D The transfer of the easement automatically occurs with the transfer of the dominant estate. It need not be expressly stated.

• B The easement from Olwen to Buck was an appurtenant easement. The back 40 acres was the dominant estate. The front 40 acres was the servient estate. An appurtenant easement is conveyed, even though it is not mentioned. by a conveyance of the dominant estate. Therefore, Sam has a valid easement over the right of way on Olwen’s land.

Page 129: 1 McLAUGHLIN MULTISTATE BAR PREPARATION COURSE Class 5 Property I Questions 1-51 Copyright 2003 Walter H. McLaughlin

129

Governing Law Rights and Obligations of Holder of Servient Estate

• The holder of an easement has the right to make repairs to property such as pipes and roads which are associated with the easement, but does not have the obligation to repair unless he agrees to do so

Page 130: 1 McLAUGHLIN MULTISTATE BAR PREPARATION COURSE Class 5 Property I Questions 1-51 Copyright 2003 Walter H. McLaughlin

130

Question 28 Degree of Difficulty Medium• July 1991 Q 191• Beach owned a tract of land called Blackacre. An

old road ran through Blackacre from the abutting public highway. The road had been used to haul wood from Blackacre. Without Beach's permission and with no initial right, Daniel, the owner of Whiteacre, which adjoined Blackacre, traveled over the old road for a period of 15 years to obtain access to Whiteacre, although Whiteacre abutted another public road. Occasionally, Daniel made repairs to the old road.

• The period of time to acquire rights by prescription in the jurisdiction is ten years.

Page 131: 1 McLAUGHLIN MULTISTATE BAR PREPARATION COURSE Class 5 Property I Questions 1-51 Copyright 2003 Walter H. McLaughlin

131

• After the expiration of 15 years, Beach conveyed a portion of Blackacre to Carrol. The deed included the following clause: "together with the right to pass and repass at all times and for all purposes over the old road." Carrol built a house fronting on the old road.

• The road was severely damaged by a spring flood, and Carrol made substantial repairs to the road. Carrol asked Daniel and Beach to contribute one-third each to the cost of repairing the flood damage. They both refused, and Carrol brought an appropriate action to compel contribution from Beach and Daniel.

• In this action, Carrol will• (A) lose as to both defendants. • (B) win as to both defendants. • (C) win as to Beach, but lose as to Daniel. • (D) win as to Daniel, but lose as to Beach.

Page 132: 1 McLAUGHLIN MULTISTATE BAR PREPARATION COURSE Class 5 Property I Questions 1-51 Copyright 2003 Walter H. McLaughlin

132

B, C and D Are Wrong A Is Right • Carrol will lose to both defendants. Carrol will lose

to Daniel because a person who acquires an easement by prescription is not burdened with any obligation to repair the easement. Carol will also lose to Beach. Carol holds an easement by grant from Beach. The owner of the servient estate, Beach, has no obligation to maintain that easement unless he explicitly agrees to do so.

Page 133: 1 McLAUGHLIN MULTISTATE BAR PREPARATION COURSE Class 5 Property I Questions 1-51 Copyright 2003 Walter H. McLaughlin

133

Governing Law Easements• When the exact location is not specified, the owner

of the servient estate has the right to locate the precise right-of-way in a reasonable manner. Once the location of that right of way was established, the easement’s location becomes fixed and cannot be moved without the consent of the owners of the servient and dominant estate.

Page 134: 1 McLAUGHLIN MULTISTATE BAR PREPARATION COURSE Class 5 Property I Questions 1-51 Copyright 2003 Walter H. McLaughlin

134

Question 29 Degree of Difficulty Hard• July 1998 Q 155• Thirty years ago Able, the then-record owner of Greenacre, a lot

contiguous to Blueacre, in fee simple, executed and delivered to Baker an instrument in writing which was denominated "Deed of Conveyance." In pertinent part it read, "Able does grant to Baker and her heirs and assigns a right-of-way for egress and ingress to Blueacre." If the quoted provision was sufficient to create an interest in land, the instrument met all other requirements for a valid grant. Baker held record title in fee simple to Blueacre, which adjoined Greenacre.

• Twelve years ago Charlie succeeded to Able's title in fee simple in Greenacre, and seven years ago Dorcas succeeded to Baker's title in fee simple in Blueacre by a deed which made no mention of a right-of-way or driveway. At the time Dorcas took title, there existed a driveway across Greenacre which showed evidence that it had been used regularly to travel between Main Road, a public road, and Blueacre.

Page 135: 1 McLAUGHLIN MULTISTATE BAR PREPARATION COURSE Class 5 Property I Questions 1-51 Copyright 2003 Walter H. McLaughlin

135

• Blueacre did have frontage on Side Road, another public road, but this means of access was seldom used because it was not as convenient to the dwelling situated on Blueacre as was Main Road. The driveway originally was established by Baker.

• Dorcas has regularly used the driveway since acquiring title. The period of time required to acquire rights by prescription in the jurisdiction is ten years.

• Six months ago Charlie notified Dorcas that Charlie planned to develop a portion of Greenacre as a residential subdivision and that Dorcas should cease any use of the driveway. After some negotiations, Charlie offered to permit Dorcas to construct another driveway to connect with the streets of the proposed subdivision. Dorcas declined this offer on the ground that travel from Blueacre to Main Road would be more circuitous.

Page 136: 1 McLAUGHLIN MULTISTATE BAR PREPARATION COURSE Class 5 Property I Questions 1-51 Copyright 2003 Walter H. McLaughlin

136

• Dorcas brought an appropriate action against Charlie to obtain a definitive adjudication of the respective rights of Dorcas and Charlie. In such lawsuit Charlie relied upon the defense that the location of the easement created by the grant from Able to Baker was governed by reasonableness and that Charlie’s proposed solution was reasonable.

• Charlie's defense should• (A) fail, because the location had been established by the

acts of Baker and Able.• (B) fail, because the location of the easement had been

fixed by prescription. • (C) prevail, because the reasonableness of Charlie's

proposal was established by Dorcas's refusal to suggest any alternative location.

• (D) prevail, because the servient owner is entitled to select the location of a right of-way if the grant fails to identify its location.

Page 137: 1 McLAUGHLIN MULTISTATE BAR PREPARATION COURSE Class 5 Property I Questions 1-51 Copyright 2003 Walter H. McLaughlin

137

B and C Are Wrong• B The right to an easement by grant across

Greenacre was conveyed by grant. There never was an adverse use of the easement area.

• C Once the location of the easement was established by the acts of Able and Baker, there was no longer an issue of reasonableness of its location or relocation. The easement area was fixed.

Page 138: 1 McLAUGHLIN MULTISTATE BAR PREPARATION COURSE Class 5 Property I Questions 1-51 Copyright 2003 Walter H. McLaughlin

138

D Is Wrong and A Is Right• The original grant of an access easement across Greenacre

to connect Blueacre with Main Street did not specify the exact location of the right of way. When the exact location is not specified, the owner of the servient estate Greenacre has the right to locate the precise right-of-way in a reasonable manner. When Able owned Greenacre and Baker owned Blueacre, the parties established the location of that right of way, and a driveway was constructed across Greenacre. Once the location of that right of way was established, the easement’s location became fixed and could not be moved without the consent of the owners of the servient and dominant estate.

Page 139: 1 McLAUGHLIN MULTISTATE BAR PREPARATION COURSE Class 5 Property I Questions 1-51 Copyright 2003 Walter H. McLaughlin

139

Governing Law Implied Easement for Light and Air

• An easement for light and air does not arise by necessity or implication.

Page 140: 1 McLAUGHLIN MULTISTATE BAR PREPARATION COURSE Class 5 Property I Questions 1-51 Copyright 2003 Walter H. McLaughlin

140

Question 30 Degree of Difficulty Easy• July 1991 Q 124• Pauline and Doris own adjacent parcels of land. On each of

their parcels was a low‑rise office building. The two office buildings were of the same height. Last year Doris decided to demolish the low-rise office building on her parcel and to erect a new high‑rise office building of substantially greater height on the parcel as permitted by the zoning and building ordinances. She secured all the governmental approvals necessary to pursue her project.

• As Doris' new building was in the course of construction, Pauline realized that the shadows it would create would place her (Pauline's) building in such deep shade that the rent she could charge for space in her building would be substantially reduced.

Page 141: 1 McLAUGHLIN MULTISTATE BAR PREPARATION COURSE Class 5 Property I Questions 1-51 Copyright 2003 Walter H. McLaughlin

141

• Pauline brought an appropriate action against Doris to enjoin the construction in order to eliminate the shadow problem and for damages. Pauline presented uncontroverted evidence that her evaluation as to the impact of the shadow on the fair rental value of her building was correct. There is no statute or ordinance (other than the building and zoning ordinances) that is applicable to the issues before the court.

• The court should• (A) grant to Pauline the requested injunction. • (B) award Pauline damages measured by the loss of

rental value, but not an injunction. • (C) grant judgment for Doris, because she had secured all

the necessary governmental approvals for the new building.

• (D) grant judgment for Doris, because Pauline has no legal right to have sunshine continue to reach the windows

of her building.

Page 142: 1 McLAUGHLIN MULTISTATE BAR PREPARATION COURSE Class 5 Property I Questions 1-51 Copyright 2003 Walter H. McLaughlin

142

A, B and C Are Wrong; D Is Right• A B Pauline has no easement right for light and air,

and will therefore not obtain an injunction, nor damages for loss of rentals.

• C The granting of government approvals would not take away private easement rights. However, in these circumstances Pauline has no private easement rights.

• D Doris will lose because there is no implied easement or easement by prescription over adjoining property for light and air.

Page 143: 1 McLAUGHLIN MULTISTATE BAR PREPARATION COURSE Class 5 Property I Questions 1-51 Copyright 2003 Walter H. McLaughlin

143

Question 31 Degree of Difficulty Easy• July 1991 Q 21• Blackacre is a large tract of land owned by a religious order known as

the Seekers. On Blackacre, the Seekers erected a large residential building where its members reside. Blackacre is surrounded by rural residential properties, and its only access to a public way is afforded by an easement over a strip of land 30-feet wide. The easement was granted to the Seekers by deed from Sally, the owner of one of the adjacent residential properties. The Seekers built a driveway on the strip, and the easement was used for 20 year without incident or objection.

• Last year, as permitted by the applicable zoning ordinance, the Seekers constructed a 200-bed nursing home and a parking lot on Blackacre, using all of Blackacre that was available for such development. The nursing home was very successful, and on Sundays visitors to the nursing home overflowed the parking facilities on Blackacre and parked all along the driveway from early in the morning through the evening hours.

Page 144: 1 McLAUGHLIN MULTISTATE BAR PREPARATION COURSE Class 5 Property I Questions 1-51 Copyright 2003 Walter H. McLaughlin

144

• After two Sundays of the resulting congestion and inconvenience, Sally erected a barrier across the driveway on Sundays preventing any use of the driveway by anyone seeking access to Blackacre. The Seekers objected.

• Sally brought an appropriate action to terminate the easement.

• The most likely result in this action is that the court will hold for

• (A) Sally, because the Seekers excessively expanded the use of the dominant tenement.

• (B) Sally, because the parking on the driveway exceeded the scope of the easement.

• (C) the Seekers, because expanded use of the easement does not terminate the easement.

• (D) the Seekers, because Sally's use of self-help denies her the right to equitable relief.

Page 145: 1 McLAUGHLIN MULTISTATE BAR PREPARATION COURSE Class 5 Property I Questions 1-51 Copyright 2003 Walter H. McLaughlin

145

A, B, and D Are Wrong; C Is Right• A The remedy when the holder of an easement overburdens that

easement is to enjoin the impermissible uses, not to terminate the easement entirely.

• B It is likely that the court would hold that the parking on the driveway exceeded the scope of the easement and would enjoin improper uses, but would not permit a barrier which prevented the easement from being used in any way.

• D The fact that the owner of an easement first resorts to self help does not constitute unclean hands or other defense to equitable relief.

• C Sally by erecting barriers prevented all use of the easement. Overuse does not destroy the easement. The Seekers possess a valid easement by grant to use the thirty-foot strip for access to their property. The easement was clearly valid for use by the original residence hall. The use of the easement to service a 200-bed nursing home is possibly valid. However, parking on the strip is probably a use which exceeds the scope of the easement originally granted. The remedy is to litigate the scope of the easement and obtain injunctive relief banning all uses which exceed the scope of the easement. Sally has no right to ban the use of the driveway for uses which are within the scope of the easement.

Page 146: 1 McLAUGHLIN MULTISTATE BAR PREPARATION COURSE Class 5 Property I Questions 1-51 Copyright 2003 Walter H. McLaughlin

146

Governing Law Prescriptive Easements

• If use is with the permission of the owner, then no prescriptive rights accrue. If nothing is said, then the use is adverse.

Page 147: 1 McLAUGHLIN MULTISTATE BAR PREPARATION COURSE Class 5 Property I Questions 1-51 Copyright 2003 Walter H. McLaughlin

147

Question 32 Degree of Difficulty Easy• Oxnard owned Goldacre, a tract of land, in fee simple. At a

time when Goldacre was in the adverse possession of Amos, Eric obtained the oral permission of Oxnard to use as a road or driveway a portion of Goldacre to reach adjoining land, Twin Pines, which Eric owned in fee simple. Thereafter, during all times relevant to this problem, Eric used this road across Goldacre regularly for ingress and egress between Twin Pines and a public highway.

• Amos quit possession of Goldacre before acquiring title by adverse possession. Without any further communication between Oxnard and Eric, Eric continued to use the road for a total period, from the time he first began to use it, sufficient to acquire an easement by prescription.

Page 148: 1 McLAUGHLIN MULTISTATE BAR PREPARATION COURSE Class 5 Property I Questions 1-51 Copyright 2003 Walter H. McLaughlin

148

• Oxnard then blocked the road and refused to permit its continued use. Eric brought suit to determine his right to continue use of the road.

• Eric should• (A) win, because his user was adverse to Amos

and, once adverse, it continued adverse until some affirmative showing of a change.

• (B) win, because Eric made no attempt to renew permission after Amos quit possession of

Goldacre. • (C) lose, because his user was with permission. • (D) lose, because there is no evidence that he

continued adverse user for the required period after Amos quit possession.

Page 149: 1 McLAUGHLIN MULTISTATE BAR PREPARATION COURSE Class 5 Property I Questions 1-51 Copyright 2003 Walter H. McLaughlin

149

A and B Are Wrong• A The original use of the property by Eric was permissive

with respect to the true owner, Oxnard. The fact that it was adverse to a Amos who was no more than a trespasser and whose adverse possession never ripened into title is not relevant on the issue of his adverse use against Eric.

• B At all times, the true owner of the property was Oxnard and Eric's use with respect to Oxnard was clearly permissive at the time he started to use the road. There is no evidence that the permissive nature of the use changed. Thus the use remains permissive until Oxnard withdraws permission or is notified that Eric is holding adversely to him. These are the only methods by which Oxnard would know that he has a cause of action to prohibit the use of the property. Failure to renew permission does not make further use adverse.

Page 150: 1 McLAUGHLIN MULTISTATE BAR PREPARATION COURSE Class 5 Property I Questions 1-51 Copyright 2003 Walter H. McLaughlin

150

D Is Wrong and C Is Right• D This choice turns on whether Eric "continued" adverse

use after Amos quit possession. Because the original use was permissive with respect to Oxnard and Eric never notified Oxnard of the change in that use, the use never was adverse as to Oxnard Therefore, it did not "continue" to be adverse once Amos left and that issue is irrelevant.

• C To obtain an easement by prescription, a person must use the land of another in an continuous, open, notorious and adverse manner for the statutory period of twenty years. In this case the initial use was permissive with respect to Oxnard. Therefore, the adverse element necessary to obtain an easement by prescription is not present.

Page 151: 1 McLAUGHLIN MULTISTATE BAR PREPARATION COURSE Class 5 Property I Questions 1-51 Copyright 2003 Walter H. McLaughlin

151

Question 33 Degree of Difficulty Hard• July 1991 Q 107• Oker owned in fee simple two adjoining lots, Lots 1 and 2.

He conveyed in fee simple Lot 1 to Frank. The deed was in usual form of a warranty deed with the following provision inserted in the appropriate place:

• "Grantor, for himself, his heirs and assigns, does covenant and agree that any reasonable expense incurred by grantee, his heirs and assigns, as the result of having to repair the retaining wall presently situated on Lot 1 at the common boundary with Lot 2, shall be reimbursed one-half the costs of repairs, and by this provision the parties intend a covenant running with the land."

Page 152: 1 McLAUGHLIN MULTISTATE BAR PREPARATION COURSE Class 5 Property I Questions 1-51 Copyright 2003 Walter H. McLaughlin

152

• Frank conveyed Lot 1 in fee simple to Sara by warranty deed in usual and regular form. The deed omitted any reference to the retaining wall or any covenant. Fifty years after Oker's conveyance to Frank, Sara conveyed Lot 1 in fee simple to Tim by warranty deed in usual form; this deed omitted any reference to the retaining wall or the covenant.There is no statute that applies to any aspect of the problems presented except a recording act and a statute providing for acquisition of title after ten years of adverse possession.

• All conveyances by deed were for a consideration equal to fair market value.

• The deed from Oker to Frank was never recorded. All other deeds were promptly and property recorded.

• Lot 2 is now owned by Henry, who took by intestate succession from Oker, now dead.

Page 153: 1 McLAUGHLIN MULTISTATE BAR PREPARATION COURSE Class 5 Property I Questions 1-51 Copyright 2003 Walter H. McLaughlin

153

• Tim expended $3,500 on the retaining wall. Then he obtained all of the original deeds in the chain from Oker to him. Shortly thereafter, Tim discovered the covenant in Oker's deed to Frank. He demanded that Henry pay $1,750, and, when Henry refused, Tim instituted an appropriate action to recover that sum from Henry. In such action, Henry asserted all defenses available to him.

• If judgment is for Henry, it will be because• (A) Tim is barred by adverse possession. • (B) Frank's deed from Oker was never recorded. • (C) Tim did not know about the covenant until after he

had incurred the expenses and, hence, could not have relied on it.

• (D) Tim's expenditures were not proved to be reasonable and customary.

Page 154: 1 McLAUGHLIN MULTISTATE BAR PREPARATION COURSE Class 5 Property I Questions 1-51 Copyright 2003 Walter H. McLaughlin

154

Outline of Transactions• Lot 1

• Oker to Frank contains covenant not recorded.

• Frank to Sara Deed recorded no mention of covenant

• 50 years after Oker-Frank deed Sara to Tim Deed Recorded; no mention of covenant

• Lot 2

• Oker to Henry by intestate succession at Oker’s Death

• Lawsuit Tim v. Henry

Page 155: 1 McLAUGHLIN MULTISTATE BAR PREPARATION COURSE Class 5 Property I Questions 1-51 Copyright 2003 Walter H. McLaughlin

155

B, C and A Are Wrong; D Is Right• B While failure to record the Oker-Frank deed would be a good

defense if the owner of lot 1 agreed to reimburse Oker or his successors, it is not a defense to Henry in his suit by Frank’s successor, Tim. The delivery of the deed from Oker to Frank containing a covenant running with the land binds Henry as Oker’s successor in title whether or not that deed is ever recorded. Henry who inherited lot 2 by intestacy is not a subsequent purchaser entitled to the protection of the recording system.

• C Tim had a valuable right to enforce a covenant even if he did not know about it when he repaired the wall. He need not rely on that right while constructing the wall to be able to sue on it.

• A No party has adversely possessed land of another during the period of time covered by this question.

• D The obligation to pay for one half of the reasonable expenses is valid and binding on Henry as the successor in title to Oker. The only way in which judgment would be in Henry’s favor is if Tim’s expenditures on the wall were not reasonable.

Page 156: 1 McLAUGHLIN MULTISTATE BAR PREPARATION COURSE Class 5 Property I Questions 1-51 Copyright 2003 Walter H. McLaughlin

156

Governing Law

• A profit a Prendre is the combination of an easement right to enter the land of another and the right to sever interests such a timber or minerals from the land and take them off of the land.

Page 157: 1 McLAUGHLIN MULTISTATE BAR PREPARATION COURSE Class 5 Property I Questions 1-51 Copyright 2003 Walter H. McLaughlin

157

Question 34 Degree of Difficulty Medium• Opus, the owner of Stoneacre, entered into a written

agreement with Miner. Under this written agreement, which was acknowledged and duly recorded, Miner, for a five-year period, was given the privilege to enter on Stoneacre to remove sand, gravel, and stone in whatever quantities Miner desired. Miner was to make monthly payments to Opus on the basis of the amount of sand, gravel, and stone removed during the previous month. Under the terms of the agreement, Miner's privilege was exclusive against all others except Opus, who reserved the right to use Stoneacre to any purpose whatsoever, including the removal of sand, gravel, and stone.

Page 158: 1 McLAUGHLIN MULTISTATE BAR PREPARATION COURSE Class 5 Property I Questions 1-51 Copyright 2003 Walter H. McLaughlin

158

• One year after the agreement was entered into, the state brought a condemnation action to take Stoneacre for a highway interchange. In the condemnation action, is Miner entitled to compensation?

• (A) Yes, because he has a license, which is a property right protected by the due process clause.

• (B) Yes, because he has a profit a prendre, which is a property right protected by the due process clause.

• (C) No, because he has a license, and licenses are not property rights protected by the due process clause.

• (D) No, because he has a profit a prendre, which is not a property right protected by the due process clause.

Page 159: 1 McLAUGHLIN MULTISTATE BAR PREPARATION COURSE Class 5 Property I Questions 1-51 Copyright 2003 Walter H. McLaughlin

159

A, C and D Are Wrong• A This agreement is more than a license because it

is not revocable. Moreover, a license is a mere contract right and not an interest in land which would require just compensation under the Eminent Domain Clause.

• C Even though the this choice correctly states that licenses are not property rights entitled to eminent domain protection, the agreement creates a profit a prendre, not a license.

• D A profit a prendre is a property right protected by the Eminent Domain Clause.

Page 160: 1 McLAUGHLIN MULTISTATE BAR PREPARATION COURSE Class 5 Property I Questions 1-51 Copyright 2003 Walter H. McLaughlin

160

B Is Right• There are two determinations necessary to correctly answer

this question. • First, what kind of right was created by the written

agreement, a license or a profit a prendre? The agreement between Opus and Miner created a profit a prendre because it was in writing and gave Miner an irrevocable right for five years to enter the land of Opus, sever some of the realty and carry it off as personalty. A license, on the other hand, need not be in writing and is revocable.

• Second, a profit a prendre is an interest in land. When such an interest, even though not a fee interest, is taken by the state, the Due Process Clause of the Fourteenth Amendment (incorporating the Fifth Amendment) entitles the owner of that right to just compensation.

Page 161: 1 McLAUGHLIN MULTISTATE BAR PREPARATION COURSE Class 5 Property I Questions 1-51 Copyright 2003 Walter H. McLaughlin

161

Governing Law Rights of Holder of a Purchase Money

Security Interest in Collateral Affixed to Real Property

• Upon default, the holder of a properly recorded purchase money security interest can remove a fixture over the despite the objection of a mortgagee of the real estate if the property subject to the interest is removable.

Page 162: 1 McLAUGHLIN MULTISTATE BAR PREPARATION COURSE Class 5 Property I Questions 1-51 Copyright 2003 Walter H. McLaughlin

162

Question 35 Degree of Difficulty Hard• July 1991 Q77 Adam owns his home, Blackacre, which

was mortgaged to Bank by a duly-recorded purchase money mortgage. Last year, Adam replaced all of Blackacre's old windows with new windows.

• Each new window consists of a window frame with three inserts: regular windows, storm windows, and screens. The windows are designed so that each insert can be easily inserted or removed from the window frame without tools to adjust to seasonal change and to facilitate the cleaning of the inserts.

• The new windows were expensive. Adam purchased them on credit, signed a financing statement, and granted a security interest in the windows to Vend, the supplier of the windows. Vend promptly and properly filed and recorded the financing statement before the windows were installed. Adam stored the old windows in the basement of Blackacre.

Page 163: 1 McLAUGHLIN MULTISTATE BAR PREPARATION COURSE Class 5 Property I Questions 1-51 Copyright 2003 Walter H. McLaughlin

163

• This year, Adam has suffered severe financial reverses and has defaulted on his mortgage obligation to Bank and on his obligation to Vend.

• Bank brought an appropriate action to enjoin Vend from its proposed repossession of the window inserts.

• In the action, the court should rule for• (A) Bank, because its mortgage was recorded first. • (B) Bank, because windows and screens, no matter their

characteristics, are an integral part of a house. • (C) Vend, because the inserts are removable. • (D) Vend, because the availability of the old windows

enables Bank to return Blackacre to its original condition. •

Page 164: 1 McLAUGHLIN MULTISTATE BAR PREPARATION COURSE Class 5 Property I Questions 1-51 Copyright 2003 Walter H. McLaughlin

164

A, B, and D Are Wrong• A Under the Uniform Commercial Code section 3-913, a

purchase money security interest in fixtures has priority over an existing mortgage with respect to those fixtures.

• B The facts indicate that the window s and screens are removable. The holder of the purchase money security interest has a right to remove them upon default even if they are an integral part of a house.

• D The essential characteristic of the windows which allows Vend to prevail is the removablility of the windows, not that the old windows are present. The only obligation of Vend is to reimburse the mortgage for damage done the property while removing the fixtures.

Page 165: 1 McLAUGHLIN MULTISTATE BAR PREPARATION COURSE Class 5 Property I Questions 1-51 Copyright 2003 Walter H. McLaughlin

165

C Is Right• UCC Section 3-913 provides that a person who

holds a valid purchase money security interest in personal property which is a fixture on real estate and who properly records that security interest can remove those fixtures from the real estate over the objection of a mortgagee whose mortgage interest predated the installation of the fixtures, provided that the fixtures are removable. In this case the purchase money security interest was properly filed, and the windows are removable.

Page 166: 1 McLAUGHLIN MULTISTATE BAR PREPARATION COURSE Class 5 Property I Questions 1-51 Copyright 2003 Walter H. McLaughlin

166

Governing Law• A tenant for a fixed term has the right to remove

personal property which he attached to the real estate at the end of his lease term, even though the property might otherwise be characterized as a fixture (real estate).

Page 167: 1 McLAUGHLIN MULTISTATE BAR PREPARATION COURSE Class 5 Property I Questions 1-51 Copyright 2003 Walter H. McLaughlin

167

Question 36 Degree of Difficulty Easy• July 1998 Q 11• A little more than five years ago, Len completed

construction of a single-family home located on Homeacre, a lot that Len owned. Five years ago, Len and Tina entered into a valid five-year written lease of Homeacre that included the following language: "This house is rented as is, without certain necessary or useful items. The parties agree that Tina may acquire and install such items as she wishes at her expense, and that she may remove them if she wishes at the termination of this lease.“

• Tina decided that the house needed, and she paid cash to have installed, standard-sized combination screen/storm windows, a freestanding refrigerator to fit a kitchen alcove built for that purpose, a built-in electric stove and oven to fit a kitchen counter opening left for that purpose, and carpeting to cover the plywood living room floor.

Page 168: 1 McLAUGHLIN MULTISTATE BAR PREPARATION COURSE Class 5 Property I Questions 1-51 Copyright 2003 Walter H. McLaughlin

168

• Last month, by legal description of the land, Len conveyed Homeacre to Pete for $100,000. Pete knew of Tina's soon-expiring tenancy, but did not examine the written lease. As the lease expiration date approached, Pete learned that Tina planned to vacate on schedule, and learned for the first time that Tina claimed and planned to remove all of the above-listed items that she had installed.

• Pete promptly brought an appropriate action to enjoin Tina from removing those items.

• The court should decide that Tina may remove• (A) none of the items. • (B) only the refrigerator. • (C) all items except the carpet. • (D) all of the items.

Page 169: 1 McLAUGHLIN MULTISTATE BAR PREPARATION COURSE Class 5 Property I Questions 1-51 Copyright 2003 Walter H. McLaughlin

169

A, B and C Are Wrong; D Is Right• A fixture is an item which once was personal property but

becomes real property when it is affixed to real estate. If such personal property is affixed to real estate by a tenant and there is no agreement with the landlord that the property will become part of the real estate and left at the end of the term, then the tenant has the right to remove those items of personal property which he attached at the end of his term providing that he repairs any damage from affixing them. The only exception to this rule is an item of personal property, such as electrical wiring, which is placed in the walls of the building and which cannot be removed without substantial damage. In this case the refrigerator was never attached and remains personal property. The storm windows, the carpet, and the oven can all be removed without significant damage to the real estate. Therefore, they are all removable trade fixtures.

Page 170: 1 McLAUGHLIN MULTISTATE BAR PREPARATION COURSE Class 5 Property I Questions 1-51 Copyright 2003 Walter H. McLaughlin

170

• Questions 37 and 38 are based on the following fact situation.

• Albert, the owner of a house and lot, leased the same to Barnes for a term of five years. In addition to the house, there was also an unattached, two-car brick garage located on the lot. Barnes earned his living as an employee in a local grocery store, but his hobby consisted of wood carving and the making of small furniture. Barnes installed a work bench, electric lights, and a radiator in the garage. He also laid pipes connecting the radiator with the heating plant inside the house.

Page 171: 1 McLAUGHLIN MULTISTATE BAR PREPARATION COURSE Class 5 Property I Questions 1-51 Copyright 2003 Walter H. McLaughlin

171

• Thereafter, Albert mortgaged the premises to Good Bank to secure a loan. Barnes was not given notice of the mortgage, but the mortgage was recorded. Still later, Albert defaulted on his mortgage payments, and Good Bank began foreclosure proceedings, as it was entitled to do under the terms of the mortgage. By this time, Barnes's lease was almost ended. Barnes began the removal of the equipment he had installed in the garage. Good Bank brought an action to enjoin the removal of the equipment mentioned above. Both Barnes and Albert were named as defendants.

Page 172: 1 McLAUGHLIN MULTISTATE BAR PREPARATION COURSE Class 5 Property I Questions 1-51 Copyright 2003 Walter H. McLaughlin

172

Question 37 Degree of Difficulty Hard• If the court refuses the injunction, it will be because

• (A) Barnes was without notice of the mortgage.

• (B) the circumstances reveal that the equipment was installed for Barnes's exclusive benefit.

• (C) in the absence of a contract agreement, a residential tenant is entitled to remove any personal property he voluntarily brings upon the premises.

• (D) the Statute of Frauds precludes Good Bank from claiming any interest in the equipment.

Page 173: 1 McLAUGHLIN MULTISTATE BAR PREPARATION COURSE Class 5 Property I Questions 1-51 Copyright 2003 Walter H. McLaughlin

173

A, C and D Are Wrong• A The mortgagee stands in no better position than the owner with

respect to the rights of a tenant to remove attached property. The tenant's knowledge of the mortgage is irrelevant because the tenant could prevail even if he knew of the mortgage.

• C This statement is not always true. There are situations where a tenant can attach personal property with the intent that it become a fixture and thereby lose title to the personalty. For example, tenant-installed electric wiring in the walls of the premises would probably be part of the real estate. If personal property is not removed in a timely fashion at the termination of the lease, it remains as part of the real estate and the tenant loses his ownership rights in the affixed property.

• D The bank's interest can only derive from its interest in the property as realty. The bank has a written mortgage which establishes that interest, and complies with the Statute of Frauds.

Page 174: 1 McLAUGHLIN MULTISTATE BAR PREPARATION COURSE Class 5 Property I Questions 1-51 Copyright 2003 Walter H. McLaughlin

174

B Is Right• When personal property is affixed to real property, the

question is whether it becomes a "fixture." If it is a fixture, its ownership is automatically transferred as the ownership of the real property is transferred. If it is not a fixture, it can be removed by the person who installed it. Whether a particular object is a fixture is primarily a question of the intention of the parties at the time it was affixed. In the leasehold situation, as here, there is a presumption that there was no intent that easily detachable property would become a fixture, and so it may be removed at the end of the leasehold term. The property involved in this question is the type that can be easily removed without damaging the real estate. Therefore, the tenant should have a right to remove it.

Page 175: 1 McLAUGHLIN MULTISTATE BAR PREPARATION COURSE Class 5 Property I Questions 1-51 Copyright 2003 Walter H. McLaughlin

175

Examsmanship note• B does not spell out the applicable legal theory

completely, but the fact that the property was installed for the tenant's exclusive benefit is the foundation for the inference that there never was an intention to leave it there permanently, and this choice comes closer than any other to setting forth the basis on which the property can be removed.

Page 176: 1 McLAUGHLIN MULTISTATE BAR PREPARATION COURSE Class 5 Property I Questions 1-51 Copyright 2003 Walter H. McLaughlin

176

Question 38 Degree of Difficulty Hard• If the equipment had been installed by Albert, but the facts

were otherwise unchanged, the effect on Good Bank's prayer for an injunction would be that the

• (A) likelihood of Good Bank's succeeding would be improved.

• (B) likelihood of Good Bank's succeeding would be lessened.

• (C) likelihood of Good Bank's succeeding would be unaffected.

• (D) outcome of the litigation would depend upon whether or not the mortgage expressly mentioned personal property located on the premises.

Page 177: 1 McLAUGHLIN MULTISTATE BAR PREPARATION COURSE Class 5 Property I Questions 1-51 Copyright 2003 Walter H. McLaughlin

177

B and C Are Wrong; A Is Right• If the property were installed by a fee owner who

planned to own the real estate indefinitely, rather than by a lessee who only had a temporary interest, the intention at the time the property was installed is much more likely that it was to be a permanent part of the real estate. Therefore, the bank, which has a security interest in the real estate, would be more likely to be able to prevent Albert from turning the fixture back into personal property and its chances of obtaining the injunction would be improved. The bank's chances of obtaining the injunction would be improved, not lessened.

Page 178: 1 McLAUGHLIN MULTISTATE BAR PREPARATION COURSE Class 5 Property I Questions 1-51 Copyright 2003 Walter H. McLaughlin

178

D Is Wrong• The outcome does not depend on whether personal

property is included in the mortgage because the bank's theory would be that the property affixed to the real estate is a permanent part of the realty which cannot be severed in order to deprive the bank of an interest in that property.

Page 179: 1 McLAUGHLIN MULTISTATE BAR PREPARATION COURSE Class 5 Property I Questions 1-51 Copyright 2003 Walter H. McLaughlin

179

Governing Law Common Scheme• If the grantor consistently imposes similar covenants

on a group of lots in a subdivision, he has created a common scheme and the owner of any lots burdened by the restrictions can sue the owner of any other lot to enforce the restrictions.

• If there is no common scheme, person’s owning lots not owned by the person imposing the restriction at the time it was imposed cannot enforce the restriction.

• Covenants running with the land can be enforced with injunctive relief.

Page 180: 1 McLAUGHLIN MULTISTATE BAR PREPARATION COURSE Class 5 Property I Questions 1-51 Copyright 2003 Walter H. McLaughlin

180

Question 39 Degree of Difficulty Hard• July 1998 Q 55• Sal owned five adjoining rectangular lots, numbered 1 through 5

inclusive, all fronting on Main Street. All of the lots are in a zone limited to one- and two-family residences under the zoning ordinance. Two years ago, Sal conveyed Lots 1, 3, and 5. None of the three deeds contained any restrictions. Each of the new owners built a one-family residence.

• One year ago, Sal conveyed Lot 2 to Peter. The deed provided that each of Peter and Sal, their respective heirs and assigns, would use Lots 2 and 4 respectively only for one-family residential purposes. The deed was promptly and properly recorded. Peter built a one family residence on Lot 2.

• Last month, Sal conveyed Lot 4 to Betty. The deed contained no restrictions. The deed from Sal to Peter was in the title report examined by Betty's lawyer. Betty obtained a building permit and commenced construction of a two family residence on Lot 4.

Page 181: 1 McLAUGHLIN MULTISTATE BAR PREPARATION COURSE Class 5 Property I Questions 1-51 Copyright 2003 Walter H. McLaughlin

181

• Peter, joined by the owners of Lots 1, 3, and 5, brought an appropriate action against Betty to enjoin the proposed use of Lot 4, or, alternatively, damages caused by Betty's breach of covenant.

• Which is the most appropriate comment concerning the outcome of this action?

• (A) All plaintiffs should be awarded their requested judgment for injunction because there was a common development scheme, but award of damages should be denied to all.

• (B) Peter should be awarded appropriate remedy, but recovery by the other plaintiffs is doubtful.

• (C) Injunction should be denied, but damages should be awarded to all plaintiffs, measured by diminution of market value, if any, suffered as a result of the proximity of Betty's two-family residence.

• (D) All plaintiffs should be denied any recovery or relief because the zoning preempts any private scheme of covenants.

Page 182: 1 McLAUGHLIN MULTISTATE BAR PREPARATION COURSE Class 5 Property I Questions 1-51 Copyright 2003 Walter H. McLaughlin

182

A, D and C Are Wrong • A D Both of these choices say that all plaintiffs are

entitled to recovery. The owners of lots 1, 3 and 5 have neither the benefit nor the burden of the restriction placed on Lots 2 and 4 and have no right to sue the owner of Lot 4 to enforce the restriction. There is no common scheme of all five lots.

• C Covenants running with the land are independent of zoning and are enforceable even though they are more restrictive than the zoning requirements.

Page 183: 1 McLAUGHLIN MULTISTATE BAR PREPARATION COURSE Class 5 Property I Questions 1-51 Copyright 2003 Walter H. McLaughlin

183

B Is Right• At the time Sal conveyed Lot 2 to Peter, he

burdened his remaining lot, Lot 4, with the same restriction he placed on Lot 4. Since these restrictions are in the form of covenants and run with the land because there is appropriate notice and they touch and concern the land, successors to Peter and Sal have the right to enforce the covenant and are burdened by it. Peter, therefore, should be awarded an appropriate remedy against Betty, Sal’s successor-in-title to Lot 4.

Page 184: 1 McLAUGHLIN MULTISTATE BAR PREPARATION COURSE Class 5 Property I Questions 1-51 Copyright 2003 Walter H. McLaughlin

184

Governing Law Covenants• Covenants run with the land and bind successor

owners if• they are in writing and formed a contract between

the original parties;• they touch and concern the land;• the burdened party has notice of the covenant, either

actual notice or constructive notice through recording;

• privity exists between the party originally imposing the restriction and the party enforcing it, and

• privity exists between the party originally burdened by the restriction and the party against whom it is being enforced.

Page 185: 1 McLAUGHLIN MULTISTATE BAR PREPARATION COURSE Class 5 Property I Questions 1-51 Copyright 2003 Walter H. McLaughlin

185

Question 40 Degree of Difficulty Hard• July 1998 Q 163• Ollie owned a large tract of land known as Peterhill. During

Ollie's lifetime, Ollie conveyed the easterly half (East Peterhill), situated in the municipality of Hawthorn, to Abel, and the westerly half (West Peterhill), situated in the municipality of Sycamore, to Betty. Each of the conveyances, which were promptly and properly recorded, contained the following language:

• “The parties agree for themselves and their heirs and assigns that the premises herein conveyed shall be used only for residential purposes; that each lot created within the premises herein conveyed shall contain not less than five acres; and that each lot shall have not more than one single-family dwelling. This agreement shall bind all successor owners of all or any portion of Peterhill and any owner of any part of Peterhill may enforce this covenant.”

Page 186: 1 McLAUGHLIN MULTISTATE BAR PREPARATION COURSE Class 5 Property I Questions 1-51 Copyright 2003 Walter H. McLaughlin

186

• After Ollie's death, Abel desired to build houses on one-half acre lots in the East Peterhill tract as authorized by current applicable zoning and building codes in Hawthorn. The area surrounding East Peterhill in Hawthorn was developed as homes built on residential community with one-half acre lots. West Peterhill was in a residential area covered by the Sycamore zoning code, which allowed residential development only on five-acre tracts of land.

• In an appropriate action brought by Betty to enjoin Abel's proposed construction on one-half acre lots, the court will find the quoted restriction to be

• (A) invalid, because of the change of circumstance in the neighborhood.

• (B) invalid, because it conflicts with the applicable zoning code. • (C) valid, but only so long as the original grantees from Ollie own

their respective tracts of Peterhill. • (D) valid, because the provision imposed an equitable servitude.

Page 187: 1 McLAUGHLIN MULTISTATE BAR PREPARATION COURSE Class 5 Property I Questions 1-51 Copyright 2003 Walter H. McLaughlin

187

A and B Are Wrong• A Courts of equity will sometimes refuse to specifically

enforce covenants which run with the land if there has been a substantial change in the neighborhood. There has not been such a change in this case. In the neighborhood abutting Abel’s land in the adjoining town houses are built on five-acre lots. The houses on one-half acre lots appear to have been built in the town of Hawthorn at the time the covenant was imposed. Therefore, there is no change of circumstances.

• B Zoning and covenants are independent methods of controlling the use of land. A covenant which is more restrictive than the applicable zoning ordinance is valid.

Page 188: 1 McLAUGHLIN MULTISTATE BAR PREPARATION COURSE Class 5 Property I Questions 1-51 Copyright 2003 Walter H. McLaughlin

188

C Is Wrong and D Is Right• As part of Ollie’s conveyance of portions of

Peterhill to Abel and Betty, the grantees agreed with Ollie and each other to restrict residential development to five-acre lots. That agreement was in writing and was recorded so that subsequent grantees would have notice of that agreement. The substance of the agreement dealing with the size of residential lots touched and concerned the land. Thus the agreement was specifically enforceable in equity, not only while the original grantees owned the property, but also by and against subsequent grantees because the agreement created an equitable servitude on all of the land.

Page 189: 1 McLAUGHLIN MULTISTATE BAR PREPARATION COURSE Class 5 Property I Questions 1-51 Copyright 2003 Walter H. McLaughlin

189

Question 41 Degree of Difficulty Medium• Fernwood Realty Company developed a residential development,

known as the Fernwood Development, which included single-family dwellings, town houses, and high-rise apartments for a total of 25,000 dwelling units. Included in the deed to each unit was a covenant under which the grantee and the grantee's “heirs and assigns” agreed to purchase electrical power only from a plant Fernwood promised to build and maintain within the development. Fernwood constructed the plant and the necessary power lines. The plant did not supply power outside the development. An appropriate and fair formula was used to determine price.

• After constructing and selling 12,500 of the units, Fernwood sold its interest in the development to Gaint Realty Investors. Gaint operated the power plant and constructed and sold the remaining 12,500 units. Each conveyance from Gaint contained the same covenant relating to electrical power that Fernwood had included in the 12,500 conveyances it had made.

Page 190: 1 McLAUGHLIN MULTISTATE BAR PREPARATION COURSE Class 5 Property I Questions 1-51 Copyright 2003 Walter H. McLaughlin

190

• Page bought a dwelling unit from Olm, who had purchased it from Fernwood. Subsequently, Page, whose lot was along the boundary of the Fernwood development, ceased buying electrical power from Gaint and began purchasing power from General Power Company, which provided such service in the area surrounding the Fernwood development. Both General Power and Gaint have governmental authorization to provide electrical services to the area. Gaint instituted an appropriate action against Page to enjoin her from obtaining electrical power from General Power.

• If judgment is for Page, it most likely will be because• (A) the covenant does not touch and concern the land. • (B) the mixture of types of residential units is viewed as

preventing one common development scheme. • (C) the covenant is a restraint on alienation. • (D) there is no privity of estate between Page and

Gaint.

Page 191: 1 McLAUGHLIN MULTISTATE BAR PREPARATION COURSE Class 5 Property I Questions 1-51 Copyright 2003 Walter H. McLaughlin

191

B, C and D Are Wrong• B A common development scheme may contain various types of use,

residential as well as commercial and industrial, as long as the areas and the scheme itself are clearly set out. Moreover, a common scheme theory is of no benefit here. A common scheme only allows restrictions to be imposed upon land that is not expressly so burdened. Here, the original deeds all clearly contained the restriction. The issue is whether successors to the lots are bound by these restrictions.

• C A covenant that imposes some restrictions on the use of the property does not prevent the owner from selling the property as to render it a restraint on alienation. Therefore, a requirement that a landowner purchase electricity at a reasonable price from one company does not amount to a restraint on alienation.

• D This is a suit to specifically enforce a covenant as an equitable servitude. The privity requirement is satisfied, since Gaint is the successor in title to Fernwood, and Page is in privity with Olm since he is Olm's successor in title through a deed.

Page 192: 1 McLAUGHLIN MULTISTATE BAR PREPARATION COURSE Class 5 Property I Questions 1-51 Copyright 2003 Walter H. McLaughlin

192

A Is Right• Gaint's action against Page is an attempt to enforce a

covenant to purchase electricity originally made by prior owners of their respective properties against a subsequent owner. If Page prevails, it is most likely because not all of those requirements to permit a covenant to run with the land have been met and thus the covenant is not enforceable against a subsequent owner, either at law or as an equitable servitude. There is a strong argument that the purchase of a fungible product like electricity, which could easily be generated hundreds of miles away, does not affect the use of the land and is not closely connected to the parcels of land owned by the plaintiff and defendant. Therefore, the touch and concern requirement for a covenant to run with the land is not met.

Page 193: 1 McLAUGHLIN MULTISTATE BAR PREPARATION COURSE Class 5 Property I Questions 1-51 Copyright 2003 Walter H. McLaughlin

193

• Questions 42-43 are based on the following fact situation.

• In 1970, Oscar, owner of a 100-acre tract, prepared and duly recorded a subdivision plan called Happy Acres. The plan showed 90 one-acre lots and a ten-acre tract in the center that was designated “Future Public School”. Oscar published and distributed a brochure promoting Happy Acres which emphasized the proximity of the lots to the school property and indicated potential tax savings “because the school district will not have to expend tax money to acquire this property.” There is no specific statute concerning the dedication of school sites.

Page 194: 1 McLAUGHLIN MULTISTATE BAR PREPARATION COURSE Class 5 Property I Questions 1-51 Copyright 2003 Walter H. McLaughlin

194

• Oscar sold 50 of the lots to individual purchasers. Each deed referred to the recorded plan and also contained the following clause: “No mobile home shall be erected on any lot within Happy Acres.” Sarah was one of the original purchasers from Oscar.

• In 1976, Oscar sold the remaining 40 lots and the ten-acre tract to Max by a deed which referred to the plan and contained the restriction relating to mobile homes. Max sold the 40 lots to individual purchasers and the ten-acre tract to Pete. None of the deeds from Max referred to the plan or contained any reference to mobile homes.

Page 195: 1 McLAUGHLIN MULTISTATE BAR PREPARATION COURSE Class 5 Property I Questions 1-51 Copyright 2003 Walter H. McLaughlin

195

Question 42 Degree of Difficulty Medium• Assume for this question only that Pete has

announced his intention of erecting a fast-food restaurant on the ten-acre tract and that Sarah has filed an action to enjoin Pete. If Sarah wins, it will be because

• (A) Sarah has an equitable servitude concerning the use of the tract.

• (B) Sarah, as a taxpayer, has legal interest in the use of the tract.

• (C) Sarah is a creditor beneficiary of Oscar's promise with respect to the tract.

• (D) Pete is not a bona fide purchaser.

Page 196: 1 McLAUGHLIN MULTISTATE BAR PREPARATION COURSE Class 5 Property I Questions 1-51 Copyright 2003 Walter H. McLaughlin

196

B and C Are Wrong • B If Sarah is going to prevent Pete from erecting a fast-

food restaurant on the ten-acre tract, she must establish an interest in the land which she derives from her ownership of a lot in the subdivision She has no standing as a taxpayer. If the municipality has an interest in the land through dedication, the municipality is the appropriate entity to enforce it.

• C Sarah is not the intended beneficiary of a promise made by Oscar to some third party, since the developer did not intend to benefit the landowners in any direct way and Sarah had no interest in her property at the time the developer promised to set aside land for the school. At best, the other lot owners are incidental beneficiaries of that promise. Moreover, if Sarah had a right to enforce such a contractual promise it might only give her a right to damages, not the injunction she seeks here.

Page 197: 1 McLAUGHLIN MULTISTATE BAR PREPARATION COURSE Class 5 Property I Questions 1-51 Copyright 2003 Walter H. McLaughlin

197

D Is Wrong and A Is Right• D The issue of whether Pete took with notice of the interest

of the municipality, i.e., whether or not he is a bona fide purchaser, is an issue for the municipality as the competing claimant to the tract, not a landowner in the subdivision, to litigate. Sarah has no standing to litigate this issue.

• A When Oscar conveyed a lot to Sarah by reference to a plan showing the school site, he agreed to burden that ten-acre tract of his remaining land by restricting its use to a school site. Since the school use was supposed to be permanent, it would appear that there was an intention that the burden of this covenant run with the land. Pete, as Oscar's successor in title, is bound by Pete's promise, which can be specifically enforced in equity.

Page 198: 1 McLAUGHLIN MULTISTATE BAR PREPARATION COURSE Class 5 Property I Questions 1-51 Copyright 2003 Walter H. McLaughlin

198

Question 43 Degree of Difficulty Medium• Assume for this question only that Joe, who purchased his

lot from Max, has placed a mobile home on it and that Sarah brings an action against Joe to force him to remove it. The result of this action will be in favor of

• (A) Sarah, because the restrictive covenant in her deed runs with the land.

• (B) Sarah, because the presence of the mobile home may adversely affect the market value of her land.

• (C) Joe, because his deed did not contain the restrictive covenant.

• (D) Joe, because he is not a direct but a remote grantee of Oscar.

Page 199: 1 McLAUGHLIN MULTISTATE BAR PREPARATION COURSE Class 5 Property I Questions 1-51 Copyright 2003 Walter H. McLaughlin

199

C and D Are Wrong• C Lots where the restriction has not been explicitly

imposed by deeds are burdened lots because those lots are part of a common scheme and the doctrine of negative reciprocal servitudes applies.

• D The covenant burdens remote as well as direct grantees because it runs with the land. A grantee must have notice for a covenant to run against him in equity. That requirement is satisfied in this case because deeds to similar lots in the subdivision have been burdened by a similar covenant, and the restriction is contained in an earlier deed in Joe's chain of title, the deed from Oscar to Max.

Page 200: 1 McLAUGHLIN MULTISTATE BAR PREPARATION COURSE Class 5 Property I Questions 1-51 Copyright 2003 Walter H. McLaughlin

200

B Is Wrong and A Is Right• B A use of one neighbor's land which reduces the value of

another's does not by itself create a right of action unless that use is so unreasonable that it constitutes a nuisance. To prevent mobile homes there must be some form of covenant running with the land which burdens the land of the defendant and benefits the land of the plaintiff.

• A The restriction against mobile homes is in writing, is contained in a deed from Oscar to Max, touches and concerns the land, and arguably is intended to bind successors in title. Thus, the restriction creates a covenant running with the land. Because the restriction is part of a common scheme, that restriction burdens Sarah’s lot and gives her the right to enforce it against other lots in the subdivision.

Page 201: 1 McLAUGHLIN MULTISTATE BAR PREPARATION COURSE Class 5 Property I Questions 1-51 Copyright 2003 Walter H. McLaughlin

201

• Question 44-45 Are Based Upon The Following Fact Situation • Owner held 500 acres in fee simple absolute. In 1960 Owner platted

and obtained all required governmental approvals of two subdivisions of 200 acres each.

• In 1960 and 1961 commercial buildings and parking facilities were constructed on one, Royal Center, in accordance with the plans disclosed by the plat for each subdivision. Royal Center continues to be used for commercial purposes.

• The plat of the other, Royal Oaks, showed 250 lots, streets, and utility and drainage easements. All of the lots in Royal Oaks were conveyed during 1960 and 1961. The deeds contained provisions, expressly stated to be binding upon the grantee, his heirs and assigns, requiring the lots to be used only for single-family, residential purposes until 1985. The deeds expressly stated that these provisions were enforceable by the owner of any lot in the Royal Oaks subdivision.

• At all times since 1949, the 200 acres of Royal Center have been zoned for shopping center use, and the 200 acres in Royal Oaks have been zoned for residential use in a classification which permits both single-family and multiple-family use.

Page 202: 1 McLAUGHLIN MULTISTATE BAR PREPARATION COURSE Class 5 Property I Questions 1-51 Copyright 2003 Walter H. McLaughlin

202

Question 44 Degree of Difficulty Easy• For this question only, assume that Owner now desires to open his remaining 100 acres as a residential subdivision of 125 lots (with appropriate streets, etc.). He has, as an essential element of his scheme, the feature that the restrictions should be identical with those he planned for the original Royal Oaks residential subdivision and, further, that lot owners in Royal Oaks should be able to enforce (by lawsuits) restrictions on the lots in the 100 acres. The zoning for the 100 acres is identical with that for the 200 acres of Royal Oaks residential subdivision. Which of the following best states the chance of success for his scheme?

• (A) He can restrict use only to the extent of that imposed by zoning (that is, to residential user by not more than four dwelling units per lot).

• (B) He cannot restrict the 100 acres to residential user because of the conflicting user for retail commercial purposes in the 200 acres comprising the shopping center.

• (C) He cannot impose any enforceable restriction to residential user only.

• (D) Any chance of success depends upon the 100 acres being considered by the courts as a part of a common development scheme which also includes the 200 acres of Royal Oaks.

Page 203: 1 McLAUGHLIN MULTISTATE BAR PREPARATION COURSE Class 5 Property I Questions 1-51 Copyright 2003 Walter H. McLaughlin

203

A, B and C Are Wrong• A This choice incorrectly states that Owner can restrict the new

development only to the extent permitted by zoning laws. Zoning laws and the imposition of restrictions by deed are separate and independent issues. Zoning is not a limitation upon the right to impose restrictions.

• B The shopping center was planned, approved and created independently of the remaining 100 acres. There is nothing in the facts that indicates the shopping center is integrally tied up with this other land. Moreover, a development scheme may include residential and commercial areas and still amount to a common scheme, as long as a definite plan is set out.

• C Restrictions on the use of land are commonly achieved with covenants such as the one described here. A restriction to residential use would be said to touch and concern the land, and, if it met other requirements, could be enforced as a covenant running with the land at law or as an equitable servitude. Such a restriction is a typical one to be enforced under a theory of common scheme. Here, it would be identical to the provisions restricting another similarly situated group of lots and will provide notice to lot owners to be restricted. This land can be restricted to residential use only.

Page 204: 1 McLAUGHLIN MULTISTATE BAR PREPARATION COURSE Class 5 Property I Questions 1-51 Copyright 2003 Walter H. McLaughlin

204

D Is Right• If the courts consider the remaining 100 acres to be part of a

common scheme which includes Royal Oaks, then the land in Royal Oaks would be legally benefited by restrictions contained in the deeds pertaining to those 100 acres. If this land is located such that it should logically be developed as part of the common scheme, some jurisdiction might include it as restricted land regardless of Owner's actions. Here, the inclusion of uniform, consistent restrictions would give rise to the expectations of the Royal Oaks owners that the new acreage is part of a common scheme and, by virtue of the language in their deeds, the new lot owners will have notice of the enforceability of these restrictions under the existing Royal Oaks development scheme. Therefore, people who own land in Royal Oaks would be allowed to enforce such restrictions.

Page 205: 1 McLAUGHLIN MULTISTATE BAR PREPARATION COURSE Class 5 Property I Questions 1-51 Copyright 2003 Walter H. McLaughlin

205

Question 45 Degree of Difficulty Medium• Which of the following devices which is reasonably

raised by the fact situation above would be most likely to implement Owner's scheme to preserve the residential character of Royal Oaks subdivision and be the most readily acceptable to purchasers?

• (A) easement

• (B) covenant

• (C) fee simple subject to a condition subsequent

• (D) zoning (assuming the appropriate authority could be persuaded to zone for single-family use only)

Page 206: 1 McLAUGHLIN MULTISTATE BAR PREPARATION COURSE Class 5 Property I Questions 1-51 Copyright 2003 Walter H. McLaughlin

206

A, C and D Are Wrong; B Is Right• A An easement is not the most appropriate device to restrict a

landowner's use of his land. It is designed to give someone other than the landowner a right to a specific use of the land.

• C A qualified fee would prevent use of the property for other than residential purposes because the holder of the fee would lose title to his land if he used it for another purpose However, the terminable nature of the fee makes the land unsalable because no bank will mortgage a property where the mortgage can lose title if he violates a use restriction

• D A zoning ordinance can ordinarily only be enforced by government authorities, not neighboring landowners. Likewise, an ordinance can be changed at will by the appropriate governmental body. Therefore there is no assurance the present zoning ordinance will remain in place.

• B Covenants will give each landowner a right to enforce the covenants which cannot be removed or changed without the consent of all landowners against neighbors without unduly damaging the marketability of the title or the ability to mortgage the property.

Page 207: 1 McLAUGHLIN MULTISTATE BAR PREPARATION COURSE Class 5 Property I Questions 1-51 Copyright 2003 Walter H. McLaughlin

207

Question 46 Degree of Difficulty Hard• Allen and Barker are equal tenants in common of a strip of

land 10 feet wide and 100 feet deep which lies between the lots on which their respective homes are situated. Both Allen and Barker need the use of the 10-foot strip as a driveway; and each fears that a new neighbor might seek partition and leave him with an unusable 5-foot strip. The best advice about how to solve their problem is

• (A) a covenant against partition. • (B) an indenture granting cross easements in the

undivided half interest of each. • (C) partition into two separate 5-foot wide strips and an

indenture granting cross easements. • (D) a trust to hold the strip in perpetuity.

Page 208: 1 McLAUGHLIN MULTISTATE BAR PREPARATION COURSE Class 5 Property I Questions 1-51 Copyright 2003 Walter H. McLaughlin

208

B and D Are Wrong• B An easement is an interest to use the land of another in a

particular way. An easement cannot be created by fee owners in property which they already own. If the dominant and servient interest in an existing easement come into the same ownership, the interests are merged and the easement is terminated. Therefore, the parties cannot effectively grant cross-easements in the undivided half interest of each.

• D The trust created would be a private trust, which would be subject to the Rule Against Perpetuities. The trust would violate the Rule because the property would be held in perpetuity and thus would not vest during lives in being plus 21 years.

Page 209: 1 McLAUGHLIN MULTISTATE BAR PREPARATION COURSE Class 5 Property I Questions 1-51 Copyright 2003 Walter H. McLaughlin

209

A Is Wrong and C Is Right• A A covenant against partition of a joint tenancy or a

tenancy in common is a restraint on alienation because it restricts the co-owner from selling an individual fee in property and requires that he forever hold the property in a co-ownership form. While the present fee owners might be able to limit their own rights to partition, the rules concerning restraints on alienation would prevent their successors in title from being bound. Therefore, this device would not succeed in accomplishing the parties’ objective.

• C Once the land is divided into two separate five-foot strips, it is possible for each party to grant an easement in the portion which he owns to the other party. The cross-easements are then appurtenant to the houselots each owns and cannot be terminated without the permission of both landowners. Easements can exist in perpetuity and, if recorded, will bind a subsequent purchaser.

Page 210: 1 McLAUGHLIN MULTISTATE BAR PREPARATION COURSE Class 5 Property I Questions 1-51 Copyright 2003 Walter H. McLaughlin

210

Governing Law Statute of Frauds

• An action for specific performance of land contract ordinarily must satisfy the Statute of Frauds. The statute is satisfied if a memorandum signed by the party to be charged contains the essential terms of the agreement.

• The parties cannot agree to waive the requirements of the Statute of Frauds.

Page 211: 1 McLAUGHLIN MULTISTATE BAR PREPARATION COURSE Class 5 Property I Questions 1-51 Copyright 2003 Walter H. McLaughlin

211

Question 47 Degree of Difficulty Medium• Taylor and Scott, an unmarried couple, purchased a

condominium as tenants in common and lived in the condominium for three years. Subsequently, they made a verbal agreement that, on the death of either of them, the survivor would own the entire condominium, and, as a result, they decided they did not need wills.

• Two years later, Taylor and Scott were involved in the same automobile accident. Taylor was killed immediately. Scott died one week later. Both died intestate. Taylor's sole heir is his brother, Mark. Scott's sole heir is her mother, Martha. Mark claimed one-half of the condominium, and Martha claimed all of it. The jurisdiction has no applicable statute except for the Statute of Frauds; nor does it recognize common-law marriages.

• In an appropriate action by Martha claiming the entire ownership of the condominium, the court will find that

Page 212: 1 McLAUGHLIN MULTISTATE BAR PREPARATION COURSE Class 5 Property I Questions 1-51 Copyright 2003 Walter H. McLaughlin

212

• (A)Martha owns the entire interest because Taylor and Scott did not make wills in reliance upon their oral agreement.

• (B)Martha owns the entire interest because she is entitled to reformation of deed to reflect the

verbal agreement. • (C)Mark and Martha each own an undivided one-

half interest because Taylor and Scott each died as the result of the same accident.

• (D)Mark and Martha each own an undivided one-half interest because the Statute of Frauds applies.

Page 213: 1 McLAUGHLIN MULTISTATE BAR PREPARATION COURSE Class 5 Property I Questions 1-51 Copyright 2003 Walter H. McLaughlin

213

A and B Are Wrong• A The agreement between Taylor and Scott is supported by

consideration, the mutual promises with respect to their interest in the property. Therefore, no reliance argument is necessary to supply the consideration. This choice is wrong because the agreement is unenforceable, and Martha will not prevail

• B The remedy of reformation is available only to correct specific types of mistakes made in the original document. Here the original deed creating the tenancy in common reflected the intention of the parties at that time. There was no mistake justifying reformation. The subsequent agreement to create a joint tenancy cannot be used for that purpose.

Page 214: 1 McLAUGHLIN MULTISTATE BAR PREPARATION COURSE Class 5 Property I Questions 1-51 Copyright 2003 Walter H. McLaughlin

214

C Is Wrong and D Is Right• The choice reaches the correct result for the wrong reason.

While the deaths resulted from the same accident, they were not simultaneous. If there was a joint tenancy, Martha as the heir of the surviving joint tenant would have prevailed. Only in the simultaneous death situation is one half of the property distributed to the heirs of each joint tenant.

• The oral agreement of Taylor and Scott to change their property interest from a tenancy in common to a joint tenancy is an agreement affecting land which is unenforceable because of the Statute of Frauds. Therefore, the parties still held the property as tenants in common and a one half interest devolves to the heirs of Scott and the heirs of Taylor.

Page 215: 1 McLAUGHLIN MULTISTATE BAR PREPARATION COURSE Class 5 Property I Questions 1-51 Copyright 2003 Walter H. McLaughlin

215

Governing Law• If there is no written agreement, a court of equity can

specifically enforce an oral agreement to convey if the part performance doctrine is satisfied. In all cases there must be an oral agreement to purchase the land which is relied upon the purchaser to his detriment.

• Part performance is satisfied in many jurisdictions if the seller has engaged in equitable fraud.

• In other jurisdictions, the court will order a conveyance only if the conduct of the parties unequivocally proves that an oral agreement to convey existed. This test is ordinarily satisfied when the purchaser pays the purchase price, has possession of the land with the permission of the seller and makes improvements on the land. This test is known as unequivocal referability.

Page 216: 1 McLAUGHLIN MULTISTATE BAR PREPARATION COURSE Class 5 Property I Questions 1-51 Copyright 2003 Walter H. McLaughlin

216

Question 48 Degree of Difficulty Easy• July 1998 Q 62• Abel owned Blackacre in fee simple. Three years ago, Abel

and Betty agreed to a month-to-month tenancy with Betty paying Abel rent each month. After six months of Betty's occupancy, Abel suggested to Betty that she could buy Blackacre for a monthly payment of no more than her rent. Abel and Betty orally agreed that Betty would pay $25,000 in cash, the annual real estate taxes, the annual fire insurance premiums, and the costs of maintaining Blackacre, plus the monthly mortgage payments that Abel owed on Blackacre.

• They further orally agreed that within six years Betty could pay whatever mortgage balances were then due and Abel would give her a warranty deed to the property. Betty's average monthly payments did turn out to be about the same as her monthly rent.

Page 217: 1 McLAUGHLIN MULTISTATE BAR PREPARATION COURSE Class 5 Property I Questions 1-51 Copyright 2003 Walter H. McLaughlin

217

• Betty fully complied with all of the obligations she had undertaken. She made some structural modifications to Blackacre. Blackacre is now worth 50% more than it was when Abel and Betty made their oral agreement. Betty made her financing arrangements and was ready to complete the purchase of Blackacre, but Abel refused to close. Betty brought an appropriate action for specific performance against Abel to enforce the agreement.

• The court should rule for• (A) Abel, because the agreements were oral and violated

the statute of frauds. • (B) Abel, subject to the return of the $25,000, because the

arrangement was still a tenancy. • (C) Betty, because the doctrine of part performance

applies. • (D) Betty, because the statute of frauds does not apply to

oral purchase and sale agreements between landlords and tenants in possession.

Page 218: 1 McLAUGHLIN MULTISTATE BAR PREPARATION COURSE Class 5 Property I Questions 1-51 Copyright 2003 Walter H. McLaughlin

218

C Is Right; A and B Are Wrong• The doctrine of part performance takes this oral

agreement out of the statute of frauds because Betty while occupying Blackacre made improvements on the property and paid a lump sum of $25,000 to Betty in reliance on the oral contract. Therefore, the statute of frauds is inapplicable and will not become applicable even if Abel returns the $25,000.

D Is Wrong• The statute of frauds applies to all contracts for the

purchase and sale of land even those between landlord and tenant. The only reason Betty wins is because of an exception to the statute where the doctrine of part performance applies.

Page 219: 1 McLAUGHLIN MULTISTATE BAR PREPARATION COURSE Class 5 Property I Questions 1-51 Copyright 2003 Walter H. McLaughlin

219

Governing Law• Liquidated damages are only collectible if the

liquidated amount is reasonable either in respect to the amount of damages which the parties anticipated at the time of making the contract, or in respect to the actual damages incurred.

Page 220: 1 McLAUGHLIN MULTISTATE BAR PREPARATION COURSE Class 5 Property I Questions 1-51 Copyright 2003 Walter H. McLaughlin

220

Question 49 Degree of Difficulty Medium• July 1998 Q 119• Ven owned Goldacre, a tract of land, in fee simple. Ven and Pur

entered into a written agreement under which Pur agreed to buy Goldacre for $100,000, its fair market value.

• The agreement contained all the essential terms of a real estate contract to sell and buy, including a date for closing. The required $50,000 down payment was made. The contract provided that in the event of Pur's breach, Ven could retain the $50,000 deposit as liquidated damages.

• Before the date set for the closing in the contract, Pur died. On the day that Addy was duly qualified as administratrix of the estate of Pur, which was after the closing date, Addy made demand for return of the $50,000 deposit.

• Ven responded by stating that he took such demand to be a declaration that Addy did not intend to complete the contract and that Ven considered the contract at an end. Ven further asserted that Ven was entitled to retain, as liquidated damages, the $50,000. The reasonable market value of Goldacre had increased to $110,000 at that time.

Page 221: 1 McLAUGHLIN MULTISTATE BAR PREPARATION COURSE Class 5 Property I Questions 1-51 Copyright 2003 Walter H. McLaughlin

221

• Addy brought an appropriate action against Ven to recover the $50,000. In answer, Ven made no affirmative claim but asserted that he was entitled to retain the $50,000 as liquidated damages as provided in the contract.

• In such lawsuit, judgment should be for• (A) Addy, because the provision relied upon by Ven is

unenforceable. • (B) Addy, because the death of Pur terminated the

contract as a matter of law. • (C) Ven, because the court should enforce the express

agreement of the contracting parties. • (D) Ven, because the doctrine of equitable conversion

prevents termination of the contract upon the death of a party.

Page 222: 1 McLAUGHLIN MULTISTATE BAR PREPARATION COURSE Class 5 Property I Questions 1-51 Copyright 2003 Walter H. McLaughlin

222

B and D Are Wrong• B The death of one of the parties to an executory

real estate contract does not terminate the contract. It is enforceable by and against the decedent’s estate.

• D The doctrine of equitable conversion only requires that the asset in the estate of the decedent be considered real estate. It does not prevent the termination of the contract. The contract is not terminated because under the law of contracts, a contract survives the death of one of the parties.

Page 223: 1 McLAUGHLIN MULTISTATE BAR PREPARATION COURSE Class 5 Property I Questions 1-51 Copyright 2003 Walter H. McLaughlin

223

C Is Wrong and A Is Right • A liquidated damage clause is enforceable only if it

is a reasonable estimate at the time the contract is made of the damage a party will suffer from a breach of contract. In this case the actual damage is the difference between the contract price and the market price at the time of closing. It is unlikely that the parties could reasonably estimate that the property would lose one half of its value between the time of the contract and the time of the closing. In fact, it gained 10% in value. Therefore, the purported liquidated damage clause is an unenforceable penalty clause.

Page 224: 1 McLAUGHLIN MULTISTATE BAR PREPARATION COURSE Class 5 Property I Questions 1-51 Copyright 2003 Walter H. McLaughlin

224

Question 50 Degree of Difficulty Hard• Seth was an elderly widower who lived alone on a small

farm which he owned. Except for the farm, including the house and its furnishings, and the usual items of personal clothing and similar things, Seth owned substantially no property. Under proper management, the farm was capable of producing an adequate family income. Because of the usual deterioration accompanying old age, Seth was unable to do farm work or even to provide for his own personal needs. Seth entered into an oral contract with his nephew, Jim, by which Seth agreed to convey the farm to Jim and Jim agreed to move into the house with Seth, operate the farm, and take care of Seth for the rest of his life.

Page 225: 1 McLAUGHLIN MULTISTATE BAR PREPARATION COURSE Class 5 Property I Questions 1-51 Copyright 2003 Walter H. McLaughlin

225

• The oral contract was silent as to when the land was to be conveyed. Jim, who lived about fifty miles away where he was operating a small business of his own, terminated his business and moved in with Seth. With the assistance of his wife, Jim gave Seth excellent care until Seth died intestate about five years after the date of the contract. In his final years Seth was confined to his bed and required much personal service of an intimate and arduous sort. Seth was survived by his only son, Sol, who was also Seth's sole heir and next of kin. Sol resided in a distant city and gave his father no attention in his father's final years. Sol showed up for Seth's funeral and demanded that Jim vacate the farm immediately. Upon Jim's refusal to do so, Sol brought an appropriate action for possession. Jim answered by way of a counterclaim to establish Jim's right to possession and title to the farm.

• If the court's decision is in favor of Jim, it will be because

Page 226: 1 McLAUGHLIN MULTISTATE BAR PREPARATION COURSE Class 5 Property I Questions 1-51 Copyright 2003 Walter H. McLaughlin

226

• (A) the land is located in a state where the Statute of Frauds will not be applied if there has been such part performance as will result in an irreparable

hardship if the contract is not performed. • (B) the land is located in a state where the Statute

of Frauds will not be applied if there has been such part performance that is by its very nature unequivocally referable to the contract.

• (C) Sol is precluded by the “clean hands” doctrine from enforcing his claim against Jim.

• (D) the blood relationship of uncle-nephew is sufficient to remove the necessity for any writing to satisfy the Statute of Frauds. –

Page 227: 1 McLAUGHLIN MULTISTATE BAR PREPARATION COURSE Class 5 Property I Questions 1-51 Copyright 2003 Walter H. McLaughlin

227

C and D Are Wrong• C Sol has done nothing which would amount to

unclean hands, and thus render him unable to seek equitable relief. Moreover, he needs no assistance from equity, since his title to the property results from his status as Seth's heir.

• D there is no exception to the Statute of Frauds which permits the enforcement of oral agreements concerning land among blood relations.

Page 228: 1 McLAUGHLIN MULTISTATE BAR PREPARATION COURSE Class 5 Property I Questions 1-51 Copyright 2003 Walter H. McLaughlin

228

B Is Wrong• There are two distinct theories under which an oral

agreement to convey land will be enforced.

• The first theory is unequivocal referability. It is used when the acts of the parties to the oral contract refer to the existence of the contract. This is not present here, because Seth continued to live on the property, and the presence of Jim on the property is as consistent with an employment relationship as it is with an agreement to transfer title.

Page 229: 1 McLAUGHLIN MULTISTATE BAR PREPARATION COURSE Class 5 Property I Questions 1-51 Copyright 2003 Walter H. McLaughlin

229

A Is Right• The second theory is that of equitable fraud. It

focuses both on the change of position of the party seeking to enforce the contract which was induced by the owner of the property, and on the hardship that the purchaser would incur if he were not able to obtain title to the land. It is basically an estoppel theory which prevents the seller from taking a position inconsistent with the promise made, upon which the buyer relied. There is such hardship and reliance here.

Page 230: 1 McLAUGHLIN MULTISTATE BAR PREPARATION COURSE Class 5 Property I Questions 1-51 Copyright 2003 Walter H. McLaughlin

230

Question 51 Degree of Difficulty Hard• Blackacre is a three-acre tract of land with a small

residence. Olga, the owner of Blackacre, rented it to Terrence at a monthly rental of $200. After Terrence had been in possession of Blackacre for several years, Terrence and Olga orally agreed that Terrence would purchase Blackacre from Olga for the sum of $24,000, payable at the rate of $200 a month for ten years and also would pay the real estate taxes and the expenses of insuring and maintaining Blackacre.

• Olga agreed to give Terrence a deed to Blackacre after five years had passed and $12,000 had been paid on account and to accept from Terrence a note secured by a mortgage for the balance.

Page 231: 1 McLAUGHLIN MULTISTATE BAR PREPARATION COURSE Class 5 Property I Questions 1-51 Copyright 2003 Walter H. McLaughlin

231

• Terrence continued in possession of Blackacre and performed his obligations as orally agreed. Terrence, without consulting Olga, made improvements for which he paid $1,000. When Terrence had paid $12,000, he tendered a proper note and mortgage to Olga and demanded the delivery of the deed as agreed. Olga did not deny the oral agreement but told Terrence that she has changed her mind, and she refused to complete the transaction. Terrence then brought an action for specific performance. Olga pleaded the Statute of Frauds as her defense.

Page 232: 1 McLAUGHLIN MULTISTATE BAR PREPARATION COURSE Class 5 Property I Questions 1-51 Copyright 2003 Walter H. McLaughlin

232

• If Olga wins, it will be because• (A) nothing Terrence could have done would have

overcome the original absence of a written agreement.

• (B) the actions and payments of Terrence are as consistent with his being a tenant as with an oral contract.

• (C) Terrence did not secure Olga's approval for the improvements that he made.

• (D) Olga has not received any unconscionable benefit, and, therefore, Terrence is not entitled to equitable relief.

Page 233: 1 McLAUGHLIN MULTISTATE BAR PREPARATION COURSE Class 5 Property I Questions 1-51 Copyright 2003 Walter H. McLaughlin

233

A and C Are Wrong• A If Terrence satisfied the requirements of the

doctrine part performance as that law is interpreted in the jurisdiction, he could have overcome the absence of a writing. In many jurisdictions he would have satisfied those requirements by paying a portion of the purchase price, going onto the land and making improvements thereon.

• C The fact that Terrence made improvements in reliance upon an oral contract might be sufficient to take the agreement out of the Statute of Frauds whether or not the improvements were made with Olga's approval.

Page 234: 1 McLAUGHLIN MULTISTATE BAR PREPARATION COURSE Class 5 Property I Questions 1-51 Copyright 2003 Walter H. McLaughlin

234

D Is Wrong• Under the equitable fraud theory of the part performance

doctrine, a court will enforce an oral agreement to convey only if the petitioner can show that undue hardship to him or unconscionable gain to his opponent will result from the court's refusal to order specific performance. An argument can be made that Olga's failure to perform under the contract has caused Terrence undue hardship in that he has performed his obligations over a five-year period in reliance on the agreement and Olga has received the benefit of the improvements to her property. On the other hand, Olga has a strong argument that $1,000 in improvements over a five-year period is not an unconscionable benefit. However, the essence of the equitable fraud theory is the harm to the purchaser, not the unconscionable benefit to the seller. Therefore, this is not Olga's best defense against the part performance doctrine.

Page 235: 1 McLAUGHLIN MULTISTATE BAR PREPARATION COURSE Class 5 Property I Questions 1-51 Copyright 2003 Walter H. McLaughlin

235

B Is Right• If the jurisdiction will permit an oral agreement to be

enforceable only where the part performance doctrine requires that the actions of the parties unequivocally refer to the existence of an oral contract to convey land, the only reason Terrence will not prevail is because his actions with respect to the property are as consistent with the continuation of his original tenancy as they are with the existence of a purchase and sale agreement. Of course, an argument can be made that payment of taxes and insurance on the property are not consistent with the terms of Terrence's tenancy, but this choice provides Olga with the best argument to overcome Terrence's argument that there has been part performance.